IMG-LOGO
Trang chủ Đánh giá năng lực ĐHQG Hồ Chí Minh Top 10 đề thi Đánh giá năng lực trường ĐHQG HCM có đáp án

Top 10 đề thi Đánh giá năng lực trường ĐHQG HCM có đáp án

Top 10 đề thi Đánh giá năng lực trường ĐHQG HCM có đáp án (Đề 4)

  • 2932 lượt thi

  • 120 câu hỏi

  • 150 phút

Danh sách câu hỏi

Câu 1:

Câu nào dưới đây không phải ca dao, dân ca?
Xem đáp án

Phương pháp giải:

Căn cứ bài cao dao, dân ca đã học.

Giải chi tiết:

- Câu: “Trong rừng có bóng trúc râm/ Trong màu xanh mát ta ngâm thơ nhàn” không phải là ca dao dân ca vì đây là câu thơ trong bài Côn Sơn ca của Nguyễn Trãi.


Câu 2:

“Nửa năm hương lửa đương nồng/ Trượng phu thoắt đã động lòng bốn phương”. Hai câu thơ trên được trích từ tác phẩm nào?
Xem đáp án

Phương pháp giải:

Căn cứ bài “Chí khí anh hùng”.

Giải chi tiết:

Hai câu thơ trên được trích từ tác phẩm “Chí khí anh hùng” nói đến đoạn Từ Hải dứt áo ra đi lập sự nghiệp anh hùng.


Câu 3:

Tác phẩm “Tương tư” của nhà thơ Nguyễn Bính được rút từ tập thơ nào?
Xem đáp án

Phương pháp giải:

Căn cứ phàn tiểu dẫn của tác phẩm “Tương tư”.

Giải chi tiết:

“Tương tư” là một trong những tác phẩm tiêu biểu của Nguyễn Bính được trích từ tập “Lỡ bước sang ngang” sáng tác năm 1939.


Câu 4:

“Ngại ngùng dợn gió e sương,/ Ngừng hoa bóng thẹn trông gương mặt dày”. Từ “ngừng” trong câu thơ được hiểu là gì?
Xem đáp án

Phương pháp giải:

Căn cứ bài Truyện Kiều.

Giải chi tiết:

- Từ “ngừng” là tiếng cổ có nghĩa là nhìn, ngắm.


Câu 5:

Câu văn sau biểu thị nghĩa tình thái nào trong các loại nghĩa tình thái đã học: “Tao không thể là người lương thiện nữa”?
Xem đáp án

Phương pháp giải:

Căn cứ vào bài Nghĩa của câu.

Giải chi tiết:

 Cụm thừ “không thể” trong câu trên chỉ khả năng xảy ra sự việc cụ thể là khả năng Chí Phèo có thể quay trở về thế giới của những người lương thiện.


Câu 6:

Hình ảnh bà Tú trong tác phẩm “Thương vợ” của Tú Xương được hiện lên như thế nào trong hai câu thơ đầu tiên?
Xem đáp án

Phương pháp giải:

Căn cứ bài “Thương vợ”.

Giải chi tiết:

Hai câu thơ đầu tác giả viết: “Quanh năm buôn bán ở mom sông/ Nuôi đủ năm con với một chồng”. Hai câu thực gợi tả cụ thể hơn cuộc sống tảo tần gắn với việc buôn bán ngược xuôi của bà Tú.


Câu 7:

Lời đề từ “Chúng thủy giai đông tẩu/ Đà giang độc Bắc lưu” trong tác phẩm Người lái đò sông Đà của Nguyễn Tuân được hiểu như thế nào?
Xem đáp án

Phương pháp giải:

Căn cứ vào phần chú thích của tác phẩm “Người lái đò sông Đà”.

Giải chi tiết:

Câu thơ: “Chúng thủy giai đông tẩu/ Đà giang độc bắc lưu” là câu thơ của tác giả Nguyễn Quang Bích khi nói đến đặc điểm khác biệt của con sông Đà theo địa lí tự nhiên. Mọi dòng sông trên đất nước Việt Nam đều chảy theo hướng đông, chỉ có sông Đà là chảy theo hướng bắc. Qua đó, Nguyễn Tuân muốn gợi mở cho người đọc vẻ đẹp đầu tiên của sông Đà. Đó là một con sông độc đáo, hung bạo.


Câu 8:

Chọn từ viết sai chính tả trong các từ sau:
Xem đáp án

Phương pháp giải:

Căn cứ các bài về chính tả.

Giải chi tiết:

- Từ viết sai chính tả là: xúi dục.

- Sửa lại: xúi giục.


Câu 9:

Xác định từ ghép tổng hợp trong các từ sau:
Xem đáp án

Phương pháp giải:

Căn bài Từ và cấu tạo từ tiếng việt.

Giải chi tiết:

- Các từ láy “lấp ló” “thấp thoáng” “mếu máo” là từ láy.

- Từ “mưa móc” là từ ghép.

“Mưa móc” là một từ cổ chỉ mưa và sương.


Câu 10:

“Anh minh hai vị thánh quân/ Sông đây rửa sạch mấy lần giáp binh/ Giặc tan muôn thuở thăng bình/ Bởi đâu đất hiểm cốt mình đức cao” (Trương Hán Siêu). “Hai vị thánh quân” được nhắc đến trong câu thơ là ai?
Xem đáp án

Phương pháp giải:

Căn cứ bài Bạch Đằng giang phú.

Giải chi tiết:

Hai vị thánh quân được nhắc đến là : Trần Thánh Tông và Trần Nhân Tông.


Câu 11:

“Dưới bóng tre của ngàn xưa, thấp thoáng mái đình, mái chùa cổ kính”, xác định chủ ngữ trong câu trên:
Xem đáp án

Phương pháp giải:

Căn cứ bài Câu trần thuật đơn không có từ là.

Giải chi tiết:

Dưới bóng tre của ngàn xưa, thấp thoáng / mái đình, mái chùa cổ kính.

                 TN                              VN                             CN

Đây là kiểu câu tồn tại, để thông báo về sự xuất hiện, tồn tại của sự vật. Một trong những cách cấu tạo câu tồn tại là đảo chủ ngữ đứng sau vị ngữ.


Câu 12:

“Một mình thì anh bạn trên trạm đỉnh Phan-xi-păng ba nghìn một trăm bốn mươi hai mét kia mới một mình hơn cháu”. “Một mình” là thành phần nào của câu.
Xem đáp án

Phương pháp giải:

Căn cứ bài Khởi ngữ.

Giải chi tiết:

- “Một mình” là thành phần khởi ngữ của câu, vì nó đứng trước chủ ngữ và nêu lên đề tài được nói đến trong câu.


Câu 13:

Trong bài ca dao sau, cụm từ nào là thành ngữ:
“Em về cắt rạ đánh tranh/ Chặt tre chẻ lạt cho anh lợp nhà/ Sớm khuya hòa thuận đôi ta/ Hơn ai gác tía lầu hoa một mình”
Xem đáp án

Phương pháp giải:

Căn cứ bài thực hành về thành ngữ, điển cố. 

Giải chi tiết:

Gác tía lầu hoa là thành ngữ dùng để chỉ cảnh sống giàu sang phú quý thời phong kiến.


Câu 14:

“Tiếng sen đã động giấc hòe,/ Bóng trăng đã xế hoa lê lại gần (Truyện Kiều, Nguyễn Du)
Từ “hoa lê” trong đoạn thơ trên được được dùng để chỉ điều gì?
Xem đáp án

Phương pháp giải:

Căn cứ bài Truyện Kiều.

Giải chi tiết:

“Hoa lê” chỉ người đẹp.


Câu 15:

Trong các câu sau:

I. Nhìn chung, văn học viết Việt Nam thời trung đại gồm hai thành phần chủ yếu là văn học chữ Hán và văn học chữ Nôm.

II. Chinh phụ ngâm, nguyên văn chữ Hán, do Đoàn Thị Điểm sáng tác.

III. Với nhân cách cao thượng, tài năng nghệ thuật trác việt, Đỗ Phủ được người Trung Quốc gọi là “Thi thánh”.

IV. “Ức Trai thi tập” “Quốc âm thi tập” và “Quân trung từ mệnh tập” là những tác phẩm viết bằng chữ Hán của Nguyễn Trãi.

Những câu nào mắc lỗi:

Xem đáp án

Phương pháp giải:

Căn cứ các tác phẩm đã học. 

Giải chi tiết:

- Chinh phụ ngâm, nguyên văn chữ Hán, do Đoàn Thị Điểm sáng tác.

=> Sai kiến thức.

=> Chinh phụ ngâm, nguyên văn chữ Hán, do Đặng Trần Côn sáng tác.

- “Ức Trai  thi tập” “Quốc âm thi tập” và “Quân trung từ mệnh tập” là những tác phẩm viết bằng chữ Hán của Nguyễn Trãi.

=> Sai kiến thức.

“Quốc âm thi tập” được sáng tác bằng chữ Nôm.


Câu 16:

Đọc đoạn trích sau và thực hiện các câu hỏi từ câu 16 đến câu 20:

Để trưởng thành, tất cả chúng ta đều phải trải qua hai cuộc đấu tranh: một cuộc đấu tranh bên ngoài và một cuộc đấu tranh ngay trong tâm trí mỗi người. Nhưng cuộc đấu tranh quan trọng nhất và có ý nghĩa nhất chính là cuộc đấu tranh diễn ra ngay trong tâm hồn mỗi người. Đó là cuộc đấu tranh chống lại những thói quen không lành mạnh, những cơn nóng giận sắp bùng phát, những lời gian dối chực trào, những phán xét thiếu cơ sở và cả những căn bệnh hiểm nghèo…. Những cuộc đấu tranh như thế diễn ra liên tục và thật sự rất gian khó, nhưng lại là điều kiện giúp bạn nhận ra cảnh giới cao nhất của mình. Hãy luôn cẩn trọng và can đảm. Hãy tiếp thu ý kiến những người xung quanh nhưng đừng để họ chi phối quá nhiều đến cuộc đời bạn. Hãy giải quyết những bất đồng trong khả năng của mình nhưng đừng quên đấu tranh đến cùng để hoàn thành mục tiêu đề ra. Đừng để bóng đen của nỗi lo sợ bao trùm đến cuộc sống của bạn. Bạn phải hiếu rằng, dù có thất bại thảm hại đến mấy chăng nữa thì bạn cũng đã học hỏi được điều gì đó bổ ích cho mình. Vì vậy, hãy tin tưởng vào con đường mình đang đi và vững vàng trong cuộc đấu tranh vì những mục tiêu cao cả. Với sự hi sinh, lòng kiên trì, quyêt tâm nỗ lực không mệt mỏi và tính tự chủ của mình, nhất định bạn sẽ thành công. Bạn chính là người làm chủ số phận của mình…”

(Trích Đánh thức khát vọng, nhiều tác giả, First News tổng hợp NXB Hồng Đức, 2017, tr.67,78)

Đoạn trích trên sử dụng phương thức biểu đạt chính nào?

Xem đáp án

Phương pháp giải:

Áp dụng kiến thức đã học về phương thức biểu đạt.

Giải chi tiết:

Đoạn văn trên được viết theo phương thức Nghị luận.


Câu 17:

Biện pháp nghệ thuật nào được sử dụng trong phần in đậm?
Xem đáp án

Phương pháp giải:

Áp dụng kiến thức về các biện pháp tu từ đã học.

Giải chi tiết:

Biện pháp điệp cấu trúc (Hãy….nhưng).


Câu 18:

Theo tác giả, cuộc đấu tranh quan trọng nhất và ý nghĩa nhất là gì?
Xem đáp án

Phương pháp giải:

Đọc, tìm ý.

Giải chi tiết:

Cuộc đấu tranh quan trọng nhất là cuộc đấu tranh diễn ra ngay bên trong tâm chí mỗi con người. “Để trưởng thành, tất cả chúng ta đều phải trải qua hai cuộc đấu tranh: một cuộc đấu tranh bên ngoài và một cuộc đấu tranh ngay trong tâm trí mỗi người. Nhưng cuộc đấu tranh quan trọng nhất và có ý nghĩa nhất chính là cuộc đấu tranh diễn ra ngay trong tâm hồn mỗi người”.


Câu 19:

Câu nói “Dù có thất bại thảm hại đến mấy chăng nữa thì bạn cũng đã học hỏi được một điều gì đó bổ ích cho mình” có ý nghĩa gì?
Xem đáp án

Phương pháp giải:

Đọc, phân tích, bình luận.

Giải chi tiết:

Nội dung câu nói: “Dù có thất bại thảm hại đến mấy chăng nữa thì bạn cũng đã học hỏi được một điều gì đó bổ ích cho mình” : Mỗi thất bại là một lần chúng ta rút ra những kinh nghiệm, những bài học xương máu cho bản thân trên con đường dẫn đến thành công. Như vậy, thất bại còn là nguồn động lực để ta không ngừng nỗ lực, cố gắng, trau dồi bản thân trở nên tốt đẹp hơn, hoàn thiện hơn.


Câu 20:

Nội dung chính của đoạn trích trên là gì?
Xem đáp án

Phương pháp giải:

Đọc, tổng hợp, khái quát nội dung chính.

Giải chi tiết:

Đoạn trích trên đưa ra hai cuộc đấu tranh diễn ra bên trong và bên ngoài mỗi con người nhưng tập trung vào cuộc đấu tranh bên trong của con người. Từ đó thúc đẩy lòng tin, sự đấu tranh vì những mục tiêu cao cả và làm chủ số phận mình.


Câu 21:

Choose a suitable word or phrase (marked A, B, C or D) to fill in each blank.

When Peter was a child, there ____ a cinema near his house.

Xem đáp án

Phương pháp giải:

Kiến thức: Thì quá khứ đơn – cấu trúc “used to”

Giải chi tiết:

Dùng “used to + V-nguyên thể” trong thì quá khứ đơn để diễn tả sự việc / thói quen trong quá khứ, hiện tại thì không còn.

Cấu trúc với “there”: there + used to be + danh từ

Phân biệt với cấu trúc “tobe/get + used to + V-ing”: quen với việc làm gì (ở hiện tại).

Tạm dịch: Khi Peter còn nhỏ, đã từng có một cái rạp chiếu phim ở gần nhà anh ấy.


Câu 22:

______ of you is going to be questioned by the police about your whereabouts at the time of the crime.
Xem đáp án

Phương pháp giải:

Kiến thức: Lượng từ

Giải chi tiết:

A. All: tất cả => All of + N số nhiều + V số nhiều

B. Each: mỗi => Each of + N / đại từ + V số ít

C. Everybody: mọi người

D. Some: một vài => Some of + N số nhiều / đại từ + V số nhiều

Sau chỗ trống, động từ tobe chia “is” => loại A, D.

“everybody” không đi với “of you” => loại C

Tạm dịch: Mỗi người trong số các em sẽ được cảnh sát thẩm vấn về nơi ở của mình vào thời điểm vụ án xảy ra.


Câu 23:

There is nothing ___________ travelling abroad.
Xem đáp án

Phương pháp giải:

Kiến thức: Cấu trúc so sánh

Giải chi tiết:

“interesting” (thú vị) là tính từ dài.

Cấu trúc so sánh với tính từ dài: tobe + more + adj dài + than …

Cấu trúc so sánh bằng với tính từ dài: tobe + as + adj dài + as …

Cấu trúc so sánh nhất với tính từ dài: tobe + the most + adj dài (+ danh từ)

D sai vì không hợp nghĩa câu: Không có gì thú vị nhất đi du lịch nước ngoài.

Tạm dịch: Không có gì thú vị hơn là đi du lịch nước ngoài.


Câu 24:

I’ll be ______ work until 5:30 p.m., but I’ll be __________ home in only 30 mnitues.
Xem đáp án

Phương pháp giải:

Kiến thức: Giới từ chỉ thời gian, nơi chốn

Giải chi tiết:

be at work: bận (làm việc)

be (at) home: ở nhà

Tạm dịch: Tôi sẽ đi làm đến 5:30 chiều, nhưng tôi sẽ ở nhà chỉ trong 30 phút nữa.


Câu 25:

The more ________ and positive you look, the better you will feel.
Xem đáp án

Phương pháp giải:

Kiến thức: Từ loại

Giải chi tiết:

Thông thường, dùng trạng từ đứng sau/trước động từ nhưng cũng có một số động từ lại cần tính từ theo sau. Ví dụ: become, get, seem, look (trông có vẻ), …

Công thức so sánh kép: The more + adj/adv + S + V, the more + adj/adv + S + V

Động từ “look” trong vế đầu mang nghĩa “trông/nhìn có vẻ” => cần điền tính từ vào chỗ trống.

A. confide (v): tin cậy

B. confidently (adv): một cách tự tin

C. confident (adj): tự tin

D. confidence (n): sự tự tin

Tạm dịch: Bạn trông càng tự tin và tích cực, bạn sẽ càng cảm thấy tốt hơn.


Câu 26:

Each of the following sentences has one error (A, B, C or D). Find it and blacken your choice on your answer sheet.
A number of people killed in traffic accidents has fallen since last month.
Xem đáp án

Phương pháp giải:

Kiến thức: Sự hòa hợp chủ - vị

Giải chi tiết:

A number of + danh từ số nhiều + động từ chia theo chủ ngữ số nhiều

The number of + danh từ số nhiều + động từ chia theo chủ ngữ số ít

Vì động từ trong câu đang chia theo chủ ngữ số ít “has fallen” => dùng “the number of”

Sửa: A number => The number

Tạm dịch: Số lượng lớn người chết vì tai nạn giao thông đã giảm kể từ tháng trước.


Câu 27:

Ellen became first woman to enter, graduate from, and teach at the Massachusetts Institute.
Xem đáp án

Phương pháp giải:

Kiến thức: Mạo từ

Giải chi tiết:

Dùng mạo từ “the” trước từ chỉ số thứ tự.

first: thứ nhất

Sửa: first => the first

Tạm dịch: Ellen trở thành người phụ nữ đầu tiên nhập học, tốt nghiệp và giảng dạy tại Viện Massachusetts.


Câu 28:

Jordan introduced us to some colleagues of him.
Xem đáp án

Phương pháp giải:

Kiến thức: Đại từ sở hữu

Giải chi tiết:

him: anh ấy => làm tân ngữ (đứng sau động từ)

“his” làm tính từ sở hữu: his + N : … của anh ấy

“his” làm đại từ sở hữu: his = his + N => có thể đứng 1 mình mà không cần danh từ theo sau

some colleagues of his = some of his colleagues: một vài đồng nghiệp của anh ấy

Sửa: him => his

Tạm dịch: Jordan giới thiệu chúng tôi với một số đồng nghiệp của anh ấy.


Câu 29:

Is it right that politicians should make important decisions without consulting the public to who they are accountable?
Xem đáp án

Phương pháp giải:

Kiến thức: Đại từ quan hệ

Giải chi tiết:

Trong mệnh đề quan hệ, dùng:

giới từ + whom => thay thế cho từ chỉ người

giới từ + which => thay thế cho từ chỉ vật

“the public” (công chúng) là từ chỉ người => dùng “giới từ + whom”

Sửa: to who => to whom

Tạm dịch: Có đúng không khi các chính trị gia nên đưa ra các quyết định quan trọng mà không hỏi ý kiến công chúng những người mà họ (các chính trị gia) phải chịu trách nhiệm với họ?


Câu 30:

In this school, mostly the children are from the Chinese community.
Xem đáp án

Phương pháp giải:

Kiến thức: Từ vựng, từ loại

Giải chi tiết:

- mostly (adv): chủ yếu là

- most (lượng từ): hầu hết, hầu như

most + N

most of + tính từ sở hữu + N

most of + the + N

- almost (adv): gần như => đứng trước động từ; hoặc: tobe + almost + danh từ

almost + all / every / no / any / nothing / no one… (gần như …)

Đứng trước chủ ngữ “the children” và “are” không thể là trạng từ => điền lượng từ

Sửa: mostly => most of

Tạm dịch: Trong ngôi trường này, hầu hết trẻ em là người gốc Hoa.


Câu 31:

Which of the following best restates each of the given sentences?
Explosives are used for catching fish and other sea animals.
Xem đáp án

Phương pháp giải:

Kiến thức: Câu bị động căn bản

Giải chi tiết:

Câu chủ động thì hiện tại đơn: S + V + O

Cấu trúc câu bị động tương ứng: O + am/is/are + P2 (by S)

Tạm dịch: Chất nổ được sử dụng để đánh bắt cá và các động vật biển khác.

A. Người ta dùng chất nổ để đánh bắt cá và các động vật biển khác.

B. Sai ngữ pháp: by + V-ing

C. Sai cấu trúc: explosives used => by using explosives

D. Cá và các động vật biển khác được đánh bắt để sử dụng chất nổ. => sai nghĩa


Câu 32:

“May I see your passport, Mrs. Scott?”, said the customs officer.
Xem đáp án

Phương pháp giải:

Kiến thức: Câu tường thuật đặc biệt

Giải chi tiết:

May I …. ? => dùng đê đưa ra yêu cầu

ask to do sth: yêu cầu làm gì (yêu cầu sự cho phép để làm điều gì đó)

suggest V-ing: gợi ý cùng làm gì

ask sb to V: yêu cầu ai làm gì

promise to V: hứa làm gì

Tạm dịch: “Tôi có thể xem hộ chiếu của bà không, bà Scott?”, nhân viên hải quan nói.

A. Nhân viên hải quan yêu cầu cho phép xem hộ chiếu của bà Scott.

B. Nhân viên hải quan đề nghị cùng xem hộ chiếu của bà Scott. => sai nghĩa

C. Nhân viên hải quan yêu cầu bà Scott xem hộ chiếu của anh ta. => sai nghĩa

D. Nhân viên hải quan hứa sẽ cho bà Scott xem hộ chiếu của anh ta. => sai nghĩa


Câu 33:

You’d better not buy that house.
Xem đáp án

Phương pháp giải:

Kiến thức: Động từ khuyết thiếu

Giải chi tiết:

‘d better + V-nguyên thể= had better + V-nguyên thể = should + V-nguyên thể: nên

had better not V-nguyên thể = should not / shouldn’t V-nguyên thể: không nên

must + V-nguyên thể: phải

couldn’t have P2: không thể nào là đã làm gì

Tạm dịch: Tốt hơn hết bạn không nên mua căn nhà đó.

A. Tốt hơn là nên mua căn nhà đó. => sai nghĩa

B. Bạn phải mua căn nhà đó. => sai nghĩa

C. Bạn không nên mua căn nhà đó.

D. Bạn không thể nào là đã mua căn nhà đó. => sai nghĩa


Câu 34:

If he could speak French, he would be able to get a job there.
Xem đáp án

Phương pháp giải:

Kiến thức: Câu điều kiện loại 2

Giải chi tiết:

Dấu hiệu: động từ trong mệnh đề điều kiện chia quá khứ đơn “could”, động từ trong mệnh đề chính chia dạng “would + V-nguyên thể”.

Cách dùng: câu điều kiện loại 2 diễn tả giả thiết không có thật ở hiện tại, dẫn đến kết quả không có thật ở hiện tại.

Trong dạng bài câu đồng nghĩa, nếu câu gốc (không phải câu điều kiện) chia ở thì hiện tại đơn mà không diễn tả sự thật, chân lý,… => dùng câu điều kiện loại 2 để viết lại.

Tạm dịch: Nếu anh ta có thể nói tiếng Pháp, anh ta sẽ có thể kiếm được một công việc ở đó.

A. Anh ấy không thể nói tiếng Pháp, vì vậy anh ấy sẽ không thể kiếm được việc làm ở đó.

B. Anh ấy không thể nói tiếng Pháp, vì vậy anh ấy đã không được giao việc làm ở đó. => dùng câu đk loại 3 để viết lại

C. Anh ấy không thể nói tiếng Pháp vì anh ấy sẽ không kiếm được việc làm ở đó. => sai nghĩa

D. Anh ấy ước mình có thể nói tiếng Pháp và làm việc ở đó. => sai nghĩa với câu gốc (anh ta đâu có ước)


Câu 35:

The more money Peter makes, the more his wife spends.
Xem đáp án

Phương pháp giải:

Kiến thức: Cấu trúc so sánh

Giải chi tiết:

Cấu trúc so sánh kép với danh từ: The more + N + S + V, the more (+ N) + S + V: càng … càng…

= S + V + more and more + N

Cấu trúc so sánh hơn với danh từ: S + V + more + N + than …

Tạm dịch: Peter càng kiếm được nhiều tiền thì vợ anh càng tiêu nhiều hơn.

A. Peter tiêu số tiền mà vợ anh ấy kiếm được. => sai nghĩa

B. Peter tiêu nhiều tiền hơn vợ. => sai nghĩa

C. Peter và vợ đều kiếm tiền. => sai nghĩa

D. Vợ của Peter ngày càng tiêu nhiều tiền hơn.


Câu 36:

Read the passage carefully.

 It has been said that life is what we make of it. In other words, if we work hard and focus on our goals, we can have great careers and enjoy high status is society. However, these opportunities don’t exist for everyone. In some places, the family you are born into will decide almost everything about your life. India‟s caste system is an example of this.

 The caste system is a major part of the Hindu religion that has existed for thousands of years. It is a way of organizing and grouping people based on the occupation of the family. Castes will determine whom people can socialize with and their place in society. Originally, a person’s caste was supposed to be determined by their personality, but over time it has been linked to their job and family.

 There are four classes, also known as varnas, in India’s caste system. The highest one is Brahmin. People in this class have jobs in education and religion. The second highest level is the Kshatriya, or ruling class. People from this group can be soldiers, landowners, or have jobs in politics. The class beneath this is the Vaishya. These people often work in the commercial sector as merchants. The fourth class level is the Shudra. Shudras typically work as unskilled labourers doing factory or farm work, or they may also be employed as artists. There is another group, the Harijan, that is at the bottom and considered to be outside of the caste system.

 Although the caste system still exists in India, the government is taking steps to improve the living conditions and decrease unemployment rates for the Shudras and Harijan. This includes providing better health care, offering literacy programmes, and making sure that people from higher social classes do not exploit them. It seems unlikely that the caste system will disappear any time soon, but the overall conditions for those at the bottom do seem to be improving.

Choose an option (A, B, C or D) that best answers each question.

The word “this” in paragraph 1 refers to ______.

Xem đáp án

Phương pháp giải:

Kiến thức: Đọc hiểu – từ thay thế

Giải chi tiết:

Từ this trong đoạn 1 chỉ ___________.

A. thực tế là nguồn gốc của bạn chủ yếu sẽ quyết định tương lai

B. niềm vui của cuộc sống ở Ấn Độ

C. chế độ đẳng cấp của Ấn Độ tồn tại hàng nghìn năm

D. phần chính của đạo Hinđu

Thông tin: In some places, the family you are born into will decide almost everything about your life. India‟s caste system is an example of this.

Tạm dịch: Ở một số nơi, gia đình mà bạn được sinh ra sẽ quyết định hầu hết mọi thứ về cuộc sống của bạn. Chế độ đẳng cấp ở Ấn Độ là một ví dụ về điều này.


Câu 37:

Which of the following is not true about India’s caste system?
Xem đáp án

Phương pháp giải:

Kiến thức: Đọc hiểu – câu hỏi chứa ‘not’

Giải chi tiết:

Điều nào sau đây KHÔNG đúng về hệ thống đẳng cấp của Ấn Độ?

A. Chế độ đẳng cấp đã được sử dụng ở Ấn Độ từ lâu đời.

B. Kshatriya là đẳng cấp cao thứ hai.

C. Cần cù giúp con người đi lên trong chế độ đẳng cấp ở Ấn Độ.

D. Có thể có một Shudra sẽ làm việc trong một trang trại.

Thông tin:

- The caste system is a major part of the Hindu religion that has existed for thousands of years.

- The second highest level is the Kshatriya, or ruling class.

- In other words, if we work hard and focus on our goals, we can have great careers and enjoy high status is society. However, these opportunities don’t exist for everyone. In some places, the family you are born into will decide almost everything about your life. India‟s caste system is an example of this.

- Shudras typically work as unskilled labourers doing factory or farm work, or they may also be employed as artists.

Tạm dịch:

- Chế độ đẳng cấp là một phần quan trọng của tôn giáo Ấn Độ đã tồn tại hàng ngàn năm nay. => A đúng

- Đẳng cấp cao thứ hai là Kshatriya, hay tầng lớp thống trị. => B đúng

- Nói cách khác, nếu chúng ta làm việc chăm chỉ và tập trung vào các mục tiêu, chúng ta có thể có sự nghiệp tuyệt vời và đạt được địa vị cao trong xã hội. Tuy nhiên, những cơ hội này không dành cho tất cả mọi người. Ở một số nơi, gia đình mà bạn được sinh ra sẽ quyết định hầu hết mọi thứ về cuộc sống của bạn. Chế độ đẳng cấp ở Ấn Độ là một ví dụ về điều này. => C sai

- Shudras thường là những người lao động không có tay nghề làm việc tại nhà máy hoặc trang trại, hoặc họ cũng có thể là những nghệ sĩ làm công. => D đúng


Câu 38:

What could replace the word “ruling” in paragraph 3?
Xem đáp án

Phương pháp giải:

Kiến thức: Đọc hiểu – từ vựng

Giải chi tiết:

Từ nào có thể thay thế cho “ruling” trong đoạn 3?

A. defeating (V-ing): đánh bại

B. guessing (V-ing): đoán

C. delaying (V-ing): trì hoãn

D. governing (adj): cai quản, quản trị

=> ruling (adj): có quyền kiểm soát, thống trị = governing

Thông tin: The second highest level is the Kshatriya, or ruling class.

Tạm dịch: Đẳng cấp cao thứ hai là Kshatriya, hay tầng lớp thống trị.


Câu 39:

What does the passage suggest about the future of the caste system?
Xem đáp án

Phương pháp giải:

Kiến thức: Đọc hiểu – suy luận

Giải chi tiết:

Đoạn văn gợi ý gì về tương lai của chế độ đẳng cấp?

A. Một ngày không xa, nó sẽ không được sử dụng ở Ấn Độ nữa.

B. Nó có thể sẽ trở nên tồi tệ hơn trước khi nó trở nên tốt hơn.

C. Các nhóm dưới cùng sẽ vươn lên thống trị các nhóm trên.

D. Nó có thể sẽ tiếp tục tồn tại lâu dài ở Ấn Độ.

Thông tin: It seems unlikely that the caste system will disappear any time soon, but the overall conditions for those at the bottom do seem to be improving.

Tạm dịch: Có vẻ như chế độ đẳng cấp sẽ không biến mất trong tương lai gần, nhưng những điều kiện chung dành cho những người ở dưới đáy dường như đang được cải thiện.


Câu 40:

Which of the following would serve as the best title for the passage?
Xem đáp án

Phương pháp giải:

Kiến thức: Đọc hiểu – ý chính

Giải chi tiết:

Cái nào sau đây sẽ là tiêu đề hay nhất cho đoạn văn?

A. Tương lai của chế độ đẳng cấp => chỉ là chi tiết đoạn cuối

B. Tìm hiểu hệ thống đẳng cấp của Ấn Độ

C. Địa vị cao trong xã hội Ấn Độ => chi tiết đoạn 1

D. Bốn giai cấp tồn tại ở Ấn Độ => chi tiết đoạn 3

Làm câu hỏi này cuối cùng, sẽ dễ dàng nhận thấy tiêu đề B phù hợp & hay nhất.


Câu 41:

Tìm tất cả các giá trị của m để phương trình \[m\sqrt {{x^2} + 2} = x + m\] có 2 nghiệm phân biệt
Xem đáp án

Phương pháp giải:

- Cô lập m, đưa phương trình về dạng \[m = f\left( x \right)\]. Khi đó số nghiệm của phương trình là số giao điểm của đồ thị hàm số \[y = f\left( x \right)\] và đường thẳng \[y = m\].

- Lập BBT của hàm số \[y = f\left( x \right)\].

- Dựa vào bảng biến thiên để xác định giá trị của m.

Giải chi tiết:

Ta có

\[{\mkern 1mu} {\mkern 1mu} {\mkern 1mu} {\mkern 1mu} {\mkern 1mu} {\mkern 1mu} {\mkern 1mu} {\mkern 1mu} m\sqrt {{x^2} + 2} = x + m \Leftrightarrow m\left( {\sqrt {{x^2} + 2} - 1} \right) = x \Leftrightarrow m = \frac{x}{{\sqrt {{x^2} + 2} - 1}} = f\left( x \right){\mkern 1mu} {\mkern 1mu} {\mkern 1mu} \left( {x \in \mathbb{R}} \right)\]

\[ \Rightarrow f'\left( x \right) = \frac{{2 - \sqrt {{x^2} + 2} }}{{{{\left( {\sqrt {{x^2} + 2} - 1} \right)}^2}}} = 0 \Leftrightarrow x = \pm \sqrt 2 \]

Bảng biến thiên:

Tìm tất cả các giá trị của m để phương trình \[m\sqrt {{x^2} + 2}  = x + m\] có 2 nghiệm phân biệt (ảnh 1)

Dựa vào bảng biến thiên ta thấy để hàm số đã cho có 2 nghiệm thì \[\left[ {\begin{array}{*{20}{l}}{ - \sqrt 2 < m < - 1}\\{1 < m < \sqrt 2 }\end{array}} \right.\].


Câu 42:

Trong mặt phẳng tọa độ, tập hợp các điểm M biểu diễn của số phức z thỏa mãn \[\left| {z + 1 + 3i} \right| = \left| {z - 2 - i} \right|\]
Xem đáp án

Phương pháp giải:

- Đặt \[z = a + bi\]. Áp dụng công thức tính môđun số phức: \[z = a + bi \Rightarrow \left| z \right| = \sqrt {{a^2} + {b^2}} \]

- Biến đổi rút ra mối quan hệ giữa \[a,{\mkern 1mu} {\mkern 1mu} b\] và suy ra quỹ tích các điểm biểu diễn số phức z.

Giải chi tiết:

Đặt \[z = a + bi{\mkern 1mu} {\mkern 1mu} \left( {a,{\mkern 1mu} {\mkern 1mu} b \in \mathbb{R}} \right).\]

Theo bài ra ta có:

\[{\mkern 1mu} {\mkern 1mu} {\mkern 1mu} {\mkern 1mu} {\mkern 1mu} {\mkern 1mu} {\mkern 1mu} \left| {z + 1 + 3i} \right| = \left| {z - 2 - i} \right| \Leftrightarrow \left| {a + bi + 1 + 3i} \right| = \left| {a + bi - 2 - i} \right|\]

\[ \Leftrightarrow {\left( {a + 1} \right)^2} + {\left( {b + 3} \right)^2} = {\left( {a - 2} \right)^2} + {\left( {b - 1} \right)^2}\]

\[ \Leftrightarrow {a^2} + 2a + 1 + {b^2} + 6b + 9 = {a^2} - 4a + 4 + {b^2} - 2b + 1\] \[ \Leftrightarrow 6a + 8b + 5 = 0\]

Suy ra tập hợp các điểm M biểu diễn số phức z là đường thẳng \[6x + 8y + 5 = 0\].

Dựa vào các đáp án ta có: Với \[A\left( { - 1; - 3} \right),{\mkern 1mu} {\mkern 1mu} B\left( {2;1} \right)\] trung điểm của đoạn AB là \[I\left( {\frac{1}{2}; - 1} \right)\].

\[\overrightarrow {AB} = \left( {3;4} \right)\] là 1 VTPT của đường trung trực của AB.

Suy ra phương trình đường trung trực của AB là:

\[3\left( {x - \frac{1}{2}} \right) + 4\left( {y + 1} \right) = 0 \Leftrightarrow 3x + 4y + \frac{5}{2} = 0 \Leftrightarrow 6x + 8y + 5 = 0\].

Vậy tập hợp điểm biểu diễn của số phức z là đường trung trực của đoạn thẳng AB.


Câu 43:

Cho tứ diện \[ABCD\] \[AB,{\mkern 1mu} {\mkern 1mu} AC,{\mkern 1mu} {\mkern 1mu} AD\] đôi một vuông góc với \[AB = 6a\], \[AC = 9a\], \[AD = 3a\]. Gọi \[M,{\mkern 1mu} {\mkern 1mu} N,{\mkern 1mu} {\mkern 1mu} P\] lần lượt là trọng tâm các tam giác \[ABC,{\mkern 1mu} {\mkern 1mu} ACD,{\mkern 1mu} {\mkern 1mu} ADB\]. Thể tích của khối tứ diện \[AMNP\] bằng:
Xem đáp án

Phương pháp giải:

- Gọi \[{M_1},{\mkern 1mu} {\mkern 1mu} {N_1},{\mkern 1mu} {\mkern 1mu} {P_1}\] lần lượt là trung điểm của \[BC,{\mkern 1mu} {\mkern 1mu} CD,{\mkern 1mu} {\mkern 1mu} BD\], sử dụng công thức tỉ lệ thể tích Simpson, so sánh \[{V_{AMNP}}\] \[{V_{A{M_1}{N_1}{P_1}}}\].

- Tiếp tục so sánh thể tích hai khối chóp có cùng chiều cao \[A.{M_1}{N_1}{P_1}\] \[A.BCD\], sử dụng tam giác đồng dạng để suy ra tỉ số diện tích hai đáy.

- Tính thể tích khối tứ diện \[ABCD\] \[{V_{ABCD}} = \frac{1}{6}AB.AC.AD\], từ đó tính được \[{V_{AMNP}}\].

Giải chi tiết:

Cho tứ diện \[ABCD\] có \[AB,{\mkern 1mu} {\mkern 1mu} AC,{\mkern 1mu} {\mkern 1mu} AD\] đôi một vuông góc với \[AB = 6a\], \[AC = 9a\], \[AD = 3a\]. Gọi \[M,{\mkern 1mu} {\mkern 1mu} N,{\mkern 1mu} {\mkern 1mu} P\] lần lượt là trọng tâm các tam giác (ảnh 1)

Gọi \[{M_1},{\mkern 1mu} {\mkern 1mu} {N_1},{\mkern 1mu} {\mkern 1mu} {P_1}\] lần lượt là trung điểm của \[BC,{\mkern 1mu} {\mkern 1mu} CD,{\mkern 1mu} {\mkern 1mu} BD\], ta có \[\frac{{AM}}{{A{M_1}}} = \frac{{AN}}{{A{N_1}}} = \frac{{AP}}{{A{P_1}}} = \frac{2}{3}\].

Khi đó \[\frac{{{V_{AMNP}}}}{{{V_{A{M_1}{N_1}{P_1}}}}} = \frac{{AM}}{{A{M_1}}}.\frac{{AN}}{{A{N_1}}}.\frac{{AP}}{{A{P_1}}} = \frac{8}{{27}}\].

Dễ thấy \[\Delta {M_1}{N_1}{P_1}\] đồng dạng với tam giác \[DBC\] theo tỉ số \[k = \frac{1}{2}\] nên \[\frac{{{S_{{M_1}{N_1}{P_1}}}}}{{{S_{DBC}}}} = \frac{1}{4}\].

Mà hai khối chóp \[A.{M_1}{N_1}{P_1}\] \[A.BCD\] có dùng chiều cao nên \[\frac{{{V_{A.{M_1}{N_1}{P_1}}}}}{{{V_{ABCD}}}} = \frac{{{S_{{M_1}{N_1}{P_1}}}}}{{{S_{DBC}}}} = \frac{1}{4}\].

Lại có \[{V_{ABCD}} = \frac{1}{6}AB.AC.AD = \frac{1}{6}.6a.9a.3a = 27{a^3}\] \[ \Rightarrow {V_{A.{M_1}{N_1}{P_1}}} = \frac{1}{4}{V_{ABCD}} = \frac{{27{a^3}}}{4}\]

Vậy \[{V_{AMNP}} = \frac{8}{{27}}{V_{A{M_1}{N_1}{P_1}}} = \frac{8}{{27}}.\frac{{27{a^3}}}{4} = 2{a^3}\].


Câu 44:

Trong không gian Oxyz, cho mặt cầu \[\left( S \right)\] tâm \[I\left( {a;b;c} \right)\] bán kính bằng 1, tiếp xúc mặt phẳng \[\left( {Oxz} \right)\]. Mệnh đề nào dưới đây đúng ?
Xem đáp án

Phương pháp giải:

Mặt cầu \[\left( S \right)\] tâm \[I\left( {a;b;c} \right)\] bán kính bằng R, tiếp xúc mặt phẳng \[\left( P \right) \Leftrightarrow d\left( {I;\left( P \right)} \right) = R\].

Giải chi tiết:

Mặt cầu \[\left( S \right)\] tâm \[I\left( {a;b;c} \right)\] bán kính bằng 1, tiếp xúc mặt phẳng \[\left( {Oxz} \right):{\mkern 1mu} {\mkern 1mu} y = 0\]

\[ \Leftrightarrow d\left( {I;\left( {Oxz} \right)} \right) = 1 \Leftrightarrow \frac{{\left| b \right|}}{1} = 1 \Leftrightarrow \left| b \right| = 1\]


Câu 45:

Đặt \[I = \int\limits_{3\sqrt 2 }^6 {\frac{{dx}}{{x\sqrt {{x^2} - 9} }}} \] \[x = \frac{3}{{\cos t}}.\] Trong các khng định sau, khẳng định nào sai?
Xem đáp án

Phương pháp giải:

Sử dụng phương pháp đổi biến và chọn đáp án đúng.

Giải chi tiết:

Ta có: \[I = \int\limits_{3\sqrt 2 }^6 {\frac{{dx}}{{x\sqrt {{x^2} - 9} }}} \]\[x = \frac{3}{{\cos t}}\]

\[ \Rightarrow dx = \frac{{ - 3{{\left( {\cos t} \right)}^\prime }}}{{{{\cos }^2}t}}dt = \frac{{3\sin t}}{{{{\cos }^2}t}}dt\] Đáp án A đúng.

Đổi cận: \[\left\{ {\begin{array}{*{20}{l}}{x = 3\sqrt 2 \Rightarrow \cos t = \frac{{\sqrt 2 }}{2} \Rightarrow t = \frac{\pi }{4}}\\{x = 6 \Rightarrow \cos t = \frac{1}{2} \Rightarrow t = \frac{\pi }{3}}\end{array}} \right.\]

\[ \Rightarrow \frac{{dx}}{{x\sqrt {{x^2} - 9} }} = \frac{1}{{\frac{3}{{\cos t}}.\sqrt {\frac{9}{{{{\cos }^2}t}} - 9} }}.\frac{{3\sin t}}{{{{\cos }^2}t}}dt\] \[ = \frac{{\sin tdt}}{{\cos t.\sqrt {\frac{{9\left( {1 - {{\cos }^2}t} \right)}}{{{{\cos }^2}t}}} }}dt = \frac{{\sin tdt}}{{3\cos t\sqrt {\frac{{{{\sin }^2}t}}{{{{\cos }^2}t}}} }}dt\]

\[ = \frac{{\sin tdt}}{{3\cos t.\tan t}}dt.\]

Đáp án B sai.


Câu 46:

Cho 19 điểm phân biệt \[{A_1},{\mkern 1mu} {\mkern 1mu} {A_2},{\mkern 1mu} {\mkern 1mu} {A_3},{\mkern 1mu} {\mkern 1mu} ...,{\mkern 1mu} {\mkern 1mu} {A_{19}}\] trong đó có 5 điểm \[{A_1},{\mkern 1mu} {\mkern 1mu} {A_2},{\mkern 1mu} {\mkern 1mu} {A_3},{\mkern 1mu} {\mkern 1mu} {A_4},{\mkern 1mu} {\mkern 1mu} {A_5}\] thẳng hàng, ngoài ra không có 3 điểm nào thẳng hàng. Hỏi có tất cả bao nhiêu tam giác có 3 đỉnh được lấy trong 19 điểm trên?
Xem đáp án

Phương pháp giải:

- Một tam giác được tạo thành từ 3 điểm không thẳng hàng, do đó tìm số bộ 3 điểm không thẳng hàng.

- Sử dụng phương pháp phần bù.

Giải chi tiết:

Chọn 3 điểm bất kì từ 19 điểm có \[C_{19}^3 = 969\] cách.

Chọn 3 điểm bất kì từ 5 điểm thẳng hàng có \[C_5^3 = 10\] cách.

⇒⇒ Số cách chọn 3 điểm không thẳng hàng là \[969 - 10 = 959\] cách.

Vậy số tam giác được tạo thành từ 19 điểm trên là 959 tam giác.


Câu 47:

Một bài trắc nghiệm có 10 câu hỏi, mỗi câu có 4 phương án lựa chọn trong đó chỉ có 1 phương án đúng. Mỗi câu đúng được 5 điểm, mỗi câu sai bị trừ 2 điểm. Một học sinh do không học bài nên đánh hú họa cho mỗi câu. Tính xác suất để học sinh đó nhận điểm dưới 1.
Xem đáp án

Phương pháp giải:

Giải chi tiết:

Xác suất để trả lời đúng 1 câu là \[\frac{1}{4}\], xác suất để trả lời sai 1 câu là \[\frac{3}{4}\].

Gọi số câu trả lời đúng là \[x{\mkern 1mu} {\mkern 1mu} \left( {0 \le x \le 10,{\mkern 1mu} {\mkern 1mu} x \in \mathbb{N}} \right)\] thì số câu trả lời sai là \[10 - x\].

Số điểm học sinh đó đạt được là \[5x - 2\left( {10 - x} \right) = 7x - 20\].

Theo giả thiết \[ \Rightarrow 7x - 20 < 1 \Leftrightarrow 7x < 21 \Leftrightarrow x < 3 \Rightarrow x \in \left\{ {0;1;2} \right\}\]

TH1: Đúng 0 câu, sai 10 câu \[{P_1} = {\left( {\frac{3}{4}} \right)^{10}}\]

TH2: Đúng 1 câu, sai 9 câu \[{P_2} = C_{10}^1.\frac{1}{4}.{\left( {\frac{3}{4}} \right)^9}\]

TH3: Đúng 2 câu, sai 8 câu \[{P_3} = C_{10}^2.{\left( {\frac{1}{4}} \right)^2}.{\left( {\frac{3}{4}} \right)^8}\]

Vậy xác suất để học sinh đó nhận điểm dưới 1 là: \[{\left( {\frac{3}{4}} \right)^{10}} + C_{10}^1.\frac{1}{4}.{\left( {\frac{3}{4}} \right)^9} + C_{10}^2.{\left( {\frac{1}{4}} \right)^2}.{\left( {\frac{3}{4}} \right)^8} \approx 0,53\].


Câu 48:

Cho 3 số thực \[a,b,c\] thỏa mãn
\[{\log _2}\left( {{{\log }_3}\left( {{{\log }_4}a} \right)} \right) = {\log _3}\left( {{{\log }_4}\left( {{{\log }_2}b} \right)} \right) = {\log _4}\left( {{{\log }_2}\left( {{{\log }_3}c} \right)} \right) = 0\]. Tính giá trị của biểu thức \[S = a + b + c\].
Xem đáp án

Phương pháp giải:

Sử dụng công thức \[{\log _a}x = n \Leftrightarrow x = {a^n}\] tìm \[a,b,c\] sau đó tính tổng S.

Giải chi tiết:

\[{\mkern 1mu} {\mkern 1mu} {\mkern 1mu} {\mkern 1mu} {\mkern 1mu} {\mkern 1mu} {\mkern 1mu} {\log _2}\left( {{{\log }_3}\left( {{{\log }_4}a} \right)} \right) = 0 \Leftrightarrow {\log _3}\left( {{{\log }_4}a} \right) = 1\]\[ \Leftrightarrow {\log _4}a = {3^1} = 1 \Leftrightarrow a = {4^3} = 64\]

\[{\mkern 1mu} {\mkern 1mu} {\mkern 1mu} {\mkern 1mu} {\mkern 1mu} {\mkern 1mu} {\mkern 1mu} {\log _3}\left( {{{\log }_4}\left( {{{\log }_2}b} \right)} \right) = 0 \Leftrightarrow {\log _4}\left( {{{\log }_2}b} \right) = 1\]\[ \Leftrightarrow {\log _2}b = {4^1} = 4 \Leftrightarrow b = {2^4} = 16\]

\[{\mkern 1mu} {\mkern 1mu} {\mkern 1mu} {\mkern 1mu} {\mkern 1mu} {\mkern 1mu} {\mkern 1mu} {\log _4}\left( {{{\log }_2}\left( {{{\log }_3}c} \right)} \right) = 0 \Leftrightarrow {\log _2}\left( {{{\log }_3}c} \right) = 1\]\[ \Leftrightarrow {\log _3}c = {2^1} = 2 \Leftrightarrow c = {3^2} = 9\]

Vậy \[S = a + b + c = 64 + 16 + 9 = 89\].


Câu 49:

Nếu Tom cho Tim 12 đô, cả hai sẽ có số tiền như nhau. Nếu Tim cho Tom 12 đô, Tom sẽ có số tiền gấp 5 lần Tim. Hỏi 3 lần số tiền của Tim nhiều hơn số tiền của Tom là bao nhiêu?
Xem đáp án

Phương pháp giải:

- Gọi x là số tiền của Tim và y là số tiền của Tom (đô) \[\left( {x;y > 0} \right)\].

- Lập 2 phương trình hai ẩn \[x;{\mkern 1mu} {\mkern 1mu} y\]

- Sử dụng phương pháp cộng đại số xác định \[3x - y\].

Giải chi tiết:

Gọi x là số tiền của Tim và y là số tiền của Tom (đồng) \[\left( {x;y > 0} \right)\].

Vì Nếu Tom cho Tim 12 đô, cả hai sẽ có số tiền như nhau nên ta có phương trình: \[x + 12 = y - 12{\mkern 1mu} {\mkern 1mu} \left( 1 \right)\]

Vì Nếu Tim cho Tom 12 đô, Tom sẽ có số tiền gấp 5 lần Tim nên ta có phương trình: \[5\left( {x - 12} \right) = y + 12{\mkern 1mu} {\mkern 1mu} {\mkern 1mu} \left( 2 \right)\]

Từ (1) và (2) ta có hệ phương trình \[\left\{ {\begin{array}{*{20}{l}}{x + 12 = y - 12}\\{5\left( {x - 12} \right) = y + 12}\end{array}} \right.\]

Cộng vế theo vế từng phương trình ta có \[6x - 48 = 2y \Leftrightarrow 6x - 2y = 48 \Leftrightarrow 3x - y = 24\].

Hỏi 3 lần số tiền của Tim nhiều hơn số tiền của Tom là 24 đô.


Câu 50:

Bạn Bình đi nhà sách và mang theo một số tiền vừa đủ mua 5 quyển tập và 3 cây viết. Nhưng khi mua, giá một quyển tập mà bạn Bình định mua đã tăng lên 800 đồng, còn giá một cây viết thì giảm đi 1000 đồng. Hỏi mua 5 quyển tập và 3 cây viết như dự định ban đầu thì bạn Bình còn dư hay thiếu bao nhiêu tiền?
Xem đáp án

Phương pháp giải:

- Gọi giá tiền 1 quyển tập và 1 cây bút ban đầu lần lượt là \[x,{\mkern 1mu} {\mkern 1mu} y{\mkern 1mu} {\mkern 1mu} \left( {x;y > 0} \right)\] (đồng).

- Tính số tiền mang theo.

- Tính số tiền thức tế đã mua, và so sánh với số tiền mang theo.

Giải chi tiết:

Gọi giá tiền 1 quyển tập và 1 cây bút ban đầu lần lượt là \[x,{\mkern 1mu} {\mkern 1mu} y{\mkern 1mu} {\mkern 1mu} \left( {x;y > 0} \right)\] (đồng).

Vì bạn Bình đi nhà sách và mang theo một số tiền vừa đủ mua 5 quyển tập và 3 cây viết nên số tiền bạn Bình mang theo là \[5x + 3y\] (đồng).

Vì khi mua, giá một quyển tập mà bạn Bình định mua đã tăng lên 800 đồng, còn giá một cây viết thì giảm đi 1000 đồng nên để mua 5 quyển tập và 3 cây viết như dự định ban đầu thì số tiền cần phải có là:

\[5\left( {x + 800} \right) + 3\left( {y - 1000} \right) = 5x + 3y + 1000\] (đồng)

Do đó số tiền để mua nhiều hơn số tiền mang đi là 1000 đồng, do đó bạn Bình cón thiếu 1000 đồng.


Câu 51:

Cho mệnh đề: “Nếu là học sinh khối 12 trường em thì đều biết bơi” là mệnh đề đúng và “là học sinh khối 12 trường em” là mệnh đề đúng. Tìm mệnh đề sai trong các đáp án sau:
Xem đáp án

Phương pháp giải:

Mệnh đề \[A \Rightarrow B\] chỉ sai khi A đúng, B sai.

Giải chi tiết:

“Nếu là học sinh khối 12 trường em thì đều biết bơi” là mệnh đề đúng và “là học sinh khối 12 trường em” là mệnh đề đúng nên mệnh đề “đều biết bơi” là đúng.

Không là học sinh khối 12 trường em” và “đều không biết bơi” là các mệnh đề sai.

Xét đáp án A: “Nếu không là học sinh lớp 12 trường em thì đều không biết bơi” là mệnh đề ĐÚNG vì “không là học sinh lớp 12” và “đều không biết bơi” là các mệnh đề sai.

Xét đáp án B: “Nếu không là học sinh lớp 12 trường em thì đều không biết bơi” là mệnh đề ĐÚNG vì “không là học sinh lớp 12” và “đều không biết bơi” là các mệnh đề sai.

Xét đáp án C: “Nếu đều không biết bơi thì là học sinh khối 12 trường em” là mệnh đề ĐÚNG vì “đều không biết bơi” là mệnh đề sai và “không là học sinh lớp 12” là mệnh đề sai.

Xét đáp án D: “Nếu là học sinh lớp 12 trường em thì đều không biết bơi” là mệnh đề SAI vì “là học sinh lớp 12” là mệnh đề đúng và “đều không biết bơi” là mệnh đề sai.


Câu 52:

Một bể trộn của một nhà máy nhận nguyên liệu lỏng từ 6 van riêng biệt được đánh nhãn: R, S, T, U, Y, Z. Mỗi một van có hại trạng thái: mở và đóng. Người điều khiển bể trộn cần đảm bảo rằng các van được đóng và mở tuân thủ theo các yêu cầu sau:

 1. Nếu T mở thì cả S và Z phải đóng

 2. R và Z không thể cùng đóng một lúc

 3. Nếu Y đóng thì Z cũng phải đóng

 4. S và U không thể cùng mở một lúc

Nếu R đóng và U mở thì điều nào sau đây buộc phải đúng?

Xem đáp án

Phương pháp giải:

Dựa vào các giả thiết, suy luận và chọn đáp án đúng.

Giải chi tiết:

R đóng suy ra Z phải mở từ điều kiện 2 (R và Z không thể cùng đóng).

Z mở suy ra T buộc phải đóng vì nếu T mở, theo điều kiện 1, cả S và Z đều phải đóng, trái ngược với điều suy ra trên.

Vì vậy phương án C là câu trả lời đúng.


Câu 53:

Dựa vào các thông tin được cung cấp dưới đây để trả lời các câu từ 53 – 56

Trong một trò chơi, có đúng 6 cái cốc úp, được xếp kể nhau thành một đường thẳng, và trong mỗi cái cốc có một quả bóng được giấu. Các cái cốc được đánh số từ 1 đến 6. Mỗi một quả bóng được sơn bằng một màu duy nhất. Màu của các quả bóng là: xanh, tím, cam, tía, đỏ và vàng. Các quả bóng được giấu dưới các cái cốc tuân theo các điều kiện sau:

- Quả bóng màu tía phải được giấu dưới cái cốc có đánh số nhỏ hơn cái cốc chứa quả bóng màu cam.

 - Quả bóng màu đỏ phải được giấu ở cái cốc kể với cái cốc chứa quả bóng máu tím.

 - Quả bóng màu xanh phải được giấu ở cốc thứ 5.

Thứ tự nào sau đây có thể là thứ tự màu của các quả bóng giấu dưới các cái cốc theo thứ tự từ 1 đến 6?

Xem đáp án

Phương pháp giải:

Dựa vào các dữ kiện đề bài cho và các đáp án để chọn đáp án đúng.

Giải chi tiết:

Vì “Quả bóng màu xanh phải được giấu ở cốc thứ 5” nên loại đáp án A và B.

Vì “Quả bóng màu tía phải được giấu dưới cái cốc có đánh số nhỏ hơn cái cốc chứa quả bóng màu cam” nên loại đáp án D.


Câu 54:

Điều nào sau đây phải đúng?
Xem đáp án

Phương pháp giải:

Dựa vào các dữ kiện đề bài cho, suy luận và chọn đáp án đúng.

Giải chi tiết:

Vì “Quả bóng màu xanh phải được giấu ở cốc thứ 5” và  “Quả bóng màu tía phải được giấu dưới cái cốc có đánh số nhỏ hơn cái cốc chứa quả bóng màu cam” nên quả bóng màu tía không thể nằm dưới cốc thứ 5 và thứ 6.

Vậy quả bóng tía nằm dưới cốc được đánh số nhỏ hươn cốc chứa quả bóng xanh.


Câu 55:

Nếu quả bóng cam nằm dưới chiếc cốc thứ hai, cặp quả bóng màu nào dưới đây có thể nằm dưới hai cốc kể nhau?
Xem đáp án

Phương pháp giải:

Dựa vào các dữ kiện đề bài cho, suy luận và chọn đáp án đúng.

Giải chi tiết:

Vì “Quả bóng màu xanh phải được giấu ở cốc thứ 5” và nếu quả bóng cam nằm dưới chiếc cốc thứ hai thì ta có bảng sau:

Nếu quả bóng cam nằm dưới chiếc cốc thứ hai, cặp quả bóng màu nào dưới đây có thể nằm dưới hai cốc kể nhau (ảnh 1)

Vì  “Quả bóng màu tía phải được giấu dưới cái cốc có đánh số nhỏ hơn cái cốc chứa quả bóng màu cam” nên quả bóng màu tía nằm ở cốc thứ nhất.

Nếu quả bóng cam nằm dưới chiếc cốc thứ hai, cặp quả bóng màu nào dưới đây có thể nằm dưới hai cốc kể nhau (ảnh 2)

Vì “Quả bóng màu đỏ phải được giấu ở cái cốc kể với cái cốc chứa quả bóng máu tím” nên quả bóng màu đỏ và bóng màu tím phải nằm ở cốc số 3 và 4 => Quả bóng màu vàng phải nằm ở cốc số 6.

Nếu quả bóng màu đỏ nằm ở cốc số 3 thì quả bóng tím nằm ở cốc thứ 4 và ngược lại, khi đó ta có:

Nếu quả bóng cam nằm dưới chiếc cốc thứ hai, cặp quả bóng màu nào dưới đây có thể nằm dưới hai cốc kể nhau (ảnh 3)

Hoặc

Nếu quả bóng cam nằm dưới chiếc cốc thứ hai, cặp quả bóng màu nào dưới đây có thể nằm dưới hai cốc kể nhau (ảnh 4)

Dựa vào các đáp án ta thấy: Cặp quả bóng có thể nằm dưới hai cốc kể nhau là Tím và Xanh.


Câu 56:

Nếu quả bóng tím nằm dưới chiếc cốc thứ nhất, cặp quả bóng màu nào dưới đây phải nằm dưới hai cốc kề nhau?
Xem đáp án

Phương pháp giải:

Dựa vào các dữ kiện đề bài cho, suy luận và chọn đáp án đúng.

Giải chi tiết:

Vì “Quả bóng màu xanh phải được giấu ở cốc thứ 5” và nếu quả bóng tím nằm dưới chiếc cốc thứ nhất ta có:

Nếu quả bóng tím nằm dưới chiếc cốc thứ nhất, cặp quả bóng màu nào dưới đây phải nằm dưới hai cốc kề nhau (ảnh 1)

Vì “Quả bóng màu đỏ phải được giấu ở cái cốc kể với cái cốc chứa quả bóng máu tím” nên quả bóng màu đỏ được giấu dưới cốc thứ 2.

Nếu quả bóng tím nằm dưới chiếc cốc thứ nhất, cặp quả bóng màu nào dưới đây phải nằm dưới hai cốc kề nhau (ảnh 2)

Vì  “Quả bóng màu tía phải được giấu dưới cái cốc có đánh số nhỏ hơn cái cốc chứa quả bóng màu cam” nên quả bóng màu tía nằm ở cốc thứ nhất => Quả bóng màu Tía không nằm dưới cốc thứ 6.

Nếu quả bóng Tía nằm dưới cốc thứ 4 => Quả bóng cam nằm ở cốc thứ 4 hoặc 6 => Quả bóng Cam và Xanh phải nằm dưới hai cốc kề nhau.

Nếu quả bóng Tía nằm dưới cốc thứ 5 => Quả bóng cam nằm ở cốc thứ 6 => Quả bóng Cam và Xanh phải nằm dưới hai cốc kề nhau.

Vậy quả bóng Cam và Xanh phải nằm dưới hai cốc kề nhau.


Câu 58:

Nếu chỉ dùng các ký tự K, L và M thì số các mật khẩu khác nhau gồm 2 ký tự là:
Xem đáp án

Phương pháp giải:

Dựa vào các giả thiết đề bài cho, suy luận và chọn đáp án đúng.

Giải chi tiết:

Vì “Nếu L xuất hiện trong mật khẩu thì nó phải xuất hiện nhiều hơn một lần” do đó nếu có ký tự L và một kí tự khác L thì mật khẩu phải có ít nhất 3 kí tự => Trái với giả thiết mật khẩu chỉ có 2 kí tự.

Do đó khi có ký tự L ta chỉ có 1 mật khẩu thỏa mãn là L L.

Vì “K không thể là ký tự đầu tiên của mật khẩu” nên ta có mật khẩu M K, tuy nhiên “Nếu K xuất hiện trong mật khẩu thì N phải xuất hiện”, mà mật khẩu lại chỉ có 2 ký tự => Mật khẩu M K không thỏa mãn.

Vậy chỉ có 1 mật khẩu thỏa mãn là L L.


Câu 59:

Từ nào sau đây là một mật khẩu hợp lệ?
Xem đáp án

Phương pháp giải:

Dựa vào các giả thiết đề bài cho và các đáp án, loại trừ và chọn đáp án đúng.

Giải chi tiết:

Vì “K không thể là ký tự đầu tiên của mật khẩu” nên loại đáp án A.

Vì “M không thể là ký tự đứng cuối hoặc kể cuối của mật khẩu” nên loại đáp án B.

Vì “O không thể là ký tự đứng cuối trừ khi L xuất hiện trong mật khẩu” nên loại đáp án D.


Câu 60:

Từ nào sau đây không là mật khẩu nhưng có thể biến đổi thành mật khẩu bằng cách thay đổi thứ tự các ký tự xuất hiện trong từ?
Xem đáp án

Phương pháp giải:

Dựa vào các giả thiết đề bài cho và các đáp án, loại trừ và chọn đáp án đúng.

Giải chi tiết:

Vì “Nếu L xuất hiện trong mật khẩu thì nó phải xuất hiện nhiều hơn một lần” nên loại đáp án A.

Các đáp án B và C đều thỏa mãn các điều kiện nên đều có thể là các mật khẩu.

Vì “M không thể là ký tự đứng cuối hoặc kể cuối của mật khẩu” nên đáp án D không là mật khẩu, tuy nhiên chỉ cần đổi vị trí của M sang các vị trí đầu tiên, thứ hai hoặc thứ ba thì ta lại được một mật khẩu. Do đó đáp án D đúng.


Câu 61:

Dựa vào các thông tin được cung cấp dưới đây để trả lời các câu từ 61 – 63

Trong một nông trường chăn nuôi bò sữa Ba Vì ta thu nhập được tài liệu sau:

Trong một nông trường chăn nuôi bò sữa Ba Vì ta thu nhập được tài liệu sau: (ảnh 1)

Số con bò cho sản lượng sữa hàng ngày cao nhất của nông trường là bao nhiêu ?

Xem đáp án

Phương pháp giải:

Quan sát bảng số liệu, xem số lượng con bò cho sản lượng cao nhất là bao nhiêu, từ đó ta chọn đáp án đúng. 

Giải chi tiết:

Sản lượng sữa hàng ngày cao nhất của một con bò là từ 15 – 17 lít sữa/ ngày.

Quan sát bảng số liệu đã cho, số con bò cho sản lượng sữa dao động trong khoảng này là: 25 con.


Câu 62:

Sản lượng sữa bình quân hàng ngày của 1 con bò là
Xem đáp án

Phương pháp giải:

Sản lượng sữa bình quân hàng ngày của mỗi con bò được tính bởi công thức \[\bar x = \frac{{\sum {{x_i}{f_i}} }}{{\sum {{f_i}} }}\]  trong đó: \[\bar x\] là sản lượng sữa bình quân của mỗi con bò, \[{x_i}\] là giá trị đại diện của lớp sản lượng.

Giá trị đại diện này được tính bằng trung điểm của đoạn (hay nửa khoảng) sản lượng.

Giải chi tiết:

Sản lượng sữa bình quân hàng ngày của 1 con bò là (ảnh 1)

Sản lượng sữa bình quân hàng ngày của mỗi con bò là:

\[\bar x = \frac{{\sum {{x_i}{f_i}} }}{{\sum {{f_i}} }} = \frac{{2516}}{{200}} = 12,58\] (lít)


Câu 63:

Số con bò cho sản lượng từ 11 – 13 lít sữa/ ngày nhiều hơn số con bò cho sản lượng sữa từ 15 – 17 lít sữa/ngày là bao nhiêu phần trăm?
Xem đáp án

Phương pháp giải:

Áp dụng công thức A nhiều hơn B số phần trăm là: \[\frac{{A - B}}{B} \times 100\% \].

Giải chi tiết:

Cho sản lượng: 11 – 13 lít sữa/ ngày : có 85 con bò.

Cho sản lượng: 15 – 17 lít sữa/ ngày: có 25 con bò.

Số con bò cho sản lượng từ 11 – 13 lít sữa/ ngày nhiều hơn số con bò cho sản lượng sữa từ 15 – 17 lít sữa/ngày số phần trăm là: \[\frac{{85 - 25}}{{25}} \times 100\% = 240\% \].


Câu 64:

Dựa vào các thông tin được cung cấp dưới đây để trả lời các câu từ 64 – 66
Để xây dựng định mức thời gian gia công một chi tiết máy người ta theo dõi quá trình gia công 25 chi tiết máy. Kết quả cho trong bảng sau:

Thời gian (phút)

14

16

18

20

24

Số chi tiết

2

6

11

4

2

 Số chi tiết máy có thời gian gia công lâu nhất là

Xem đáp án

Phương pháp giải:

Quan sát bảng số liệu, lấy thông tin thời gian gia công lâu nhất của một chi tiết máy, xem có bao nhiêu chi tiết máy có thời gian gia công đó. Chọn đáp án đúng.

Giải chi tiết:

Thời gian gia công lâu nhất của một chi tiết máy là: 24 phút

Số chi tiết máy có thời gian gia công lâu nhất là: 2 chi tiết máy.


Câu 65:

Thời gian gia công 25 chi tiết máy đã cho (không tính thời gian nghỉ) là:
Xem đáp án

Phương pháp giải:

Tính tổng thời gian gia công 25 chi tiết máy.

Giải chi tiết:

Thời gian gia công 25 chi tiết máy (không tính thời gian nghỉ) là: \[14.2 + 16.6 + 18.11 + 20.4 + 24.2 = 450\]  (phút)


Câu 66:

Thời gian gia công trung bình một chi tiết máy là ….. phút
Xem đáp án

Phương pháp giải:

Áp dụng công thức tính trung bình cộng \[\bar X = \frac{{{x_1}{n_1} + {x_2}{n_2} + ... + {x_n}{n_n}}}{n}\] với \[{n_1} + {n_2} + ... + {n_n} = n\].

Giải chi tiết:

Thời gian gia công trung bình một chi tiết máy là: \[\bar X = \frac{{14.2 + 16.6 + 18.11 + 20.4 + 24.2}}{{25}} = 18\] (phút)


Câu 67:

Dựa vào các thông tin được cung cấp dưới đây để trả lời các câu từ 67 – 70
Có tài liệu về doanh thu tiêu thụ sản phẩm của doanh nghiệp bánh kẹo Hải Hà như sau:

Có tài liệu về doanh thu tiêu thụ sản phẩm của doanh nghiệp bánh kẹo Hải Hà như sau: (ảnh 1)

Doanh thu thực tế của sản phẩm Bánh quy trong năm 2007 là bao nhiêu?

Xem đáp án

Phương pháp giải:

Tỷ lệ thực hiện kế hoạch = Doanh thu thực tế (DTTT) : Doanh thu kế hoạch (DTKH) .100%

\[ \Rightarrow DTTT = \] tỉ lệ thực hiện kế hoạch về doanh thu \[ \times DTKH\]

Giải chi tiết:

Doanh thu thực thực tế của Bánh quy trong năm 2007 là: \[1200.110\% = 1320\] (triệu đồng)


Câu 68:

Tính tỷ lệ thực hiện kế hoạch về doanh thu tiêu thụ sản phẩm của doanh nghiệp bánh kẹo Hải Hà trong năm 2007.
Xem đáp án

Phương pháp giải:

Tính tổng doanh thu thực tế, tổng doanh thu theo kế hoạch trong năm 2007, rồi tính tỉ lệ của chúng.

Giải chi tiết:

Tổng doanh thu theo kế hoạch của doanh nghiệp bánh kẹo Hải Hà trong năm 2007 là:

\[1200 + 3400 + 1600 = 6200\] (triệu đồng)

Tổng doanh thu thực tế của doanh nghiệp bánh kẹo Hải Hà trong năm 2007 là:

\[1200.110\% + 3400.105\% + 1600.102\% = 6522\] (triệu đồng)

Tính tỷ lệ thực hiện kế hoạch về doanh thu tiêu thụ sản phẩm của doanh nghiệp bánh kẹo Hải Hà trong năm 2007 là: \[\frac{{6522}}{{6200}}.100\% \approx 105,2\% \].


Câu 69:

Tổng doanh thu theo kế hoạch năm 2008 của doanh nghiệp là bao nhiêu?
Xem đáp án

Phương pháp giải:

Tính doanh thu kế hoạch của từng loại sản phẩm: bánh quy, kẹo mềm, thạch dừa. Sau đó tính tổng ba loại doanh thu này theo kế hoạch.

Giải chi tiết:

Doanh thu kế hoạch của sản phẩm Bánh quy năm 2008 là: \[1400:112 \times 100 = 1250\] (triệu đồng)

Doanh thu kế hoạch của sản phẩm Kẹo mềm năm 2008 là: \[3620:110 \times 100 \approx 3290,91\] (triệu đồng)

Doanh thu kế hoạch của sản phẩm Thạch dừa năm 2008 là: \[1800:105 \times 100 \approx 1714,29\] (triệu đồng)

Tổng doanh thu theo kế hoạch năm 2008 của doanh nghiệp là:

\[1250 + 3290,91 + 1714,29 = 6255,2\] (triệu đồng) = 6, 2552 tỉ đồng.


Câu 70:

Tính tỷ lệ thực hiện kế hoạch về doanh thu tiêu thụ sản phẩm của doanh nghiệp bánh kẹo Hải Hà trong cả hai năm.
Xem đáp án

Phương pháp giải:

Tỷ lệ thực hiện kế hoạch cả 2 năm = Doanh thu thực tế (DTTT) của 2 năm : Doanh thu kế hoạch (DTKH) của cả 2 năm .100%.

Giải chi tiết:

Tổng doanh thu thực tế của doanh nghiệp bánh kẹo Hải Hà trong năm 2007 là: 6522 triệu đồng (câu 68).

Tổng doanh thu theo kế hoạch của doanh nghiệp bánh kẹo Hải Hà trong năm 2007 là: 6200 triệu đồng (câu 68).

Tổng doanh thu thực tế của doanh nghiệp năm 2008 là: \[1400 + 3620 + 1800 = 6820\] (triệu đồng)

Tổng doanh thu theo kế hoạch năm 2008 của doanh nghiệp là: 6255,2 triệu đồng (câu 69).

Tỷ lệ thực hiện kế hoạch về doanh thu tiêu thụ sản phẩm của doanh nghiệp bánh kẹo Hải Hà trong cả hai năm 2007 và 2008 là: \[\frac{{6522 + 6820}}{{6200 + 6255,2}}.100\% \approx 107,12\% \]


Câu 71:

Anion \[X{Y_3}^{2 - }\] có tổng số hạt mang điện là 62 hạt. Số hạt mang điện trong hạt nhân của Y nhiều hơn số hạt mang điện trong hạt nhân của X là 2 hạt. Biết số hiệu nguyên tử của một số nguyên tố như sau: ZC = 6; ZN = 7; ZO = 8; ZP = 15; ZO = 16; ZCl = 17. Nhận định nào sau đây sai?
Xem đáp án

Phương pháp giải:

Gọi số hạt proton và electron của nguyên tử nguyên tố X là px và ex (px, ex N*)

Gọi số hạt proton và electron của nguyên tử nguyên tố Y là py và ey (py, ey N*)

Dựa vào tổng số hạt mang điện trong anion và sự chênh lệch số hạt mang điện trong hạt nhân giữa X và Y

px và py Nguyên tố X và Y.

Giải chi tiết:

Gọi số hạt proton và electron của nguyên tử nguyên tố X là px và ex (px, ex N*) px = ex

Gọi số hạt proton và electron của nguyên tử nguyên tố Y là py và ey (py, ey N*) py = ey

* Anion \[X{Y_3}^{2 - }\] có tổng số hạt mang điện là 62 hạt.

px + ex + 3(py + ey)+ 2 = 62

2px + 6py = 60 (1)

* Số hạt mang điện trong hạt nhân của Y nhiều hơn số hạt mang điện trong hạt nhân của X là 2 hạt.

py – px = 2 (2)

* Từ (1) và (2) \[\left\{ {\begin{array}{*{20}{l}}{{p_x} = 6}\\{{p_y} = 8}\end{array}} \right.\]

Vậy X là C và Y là O.

* Xét các nhận định:

(A) đúng vì Y là O (thuộc chu kì 2, nhóm VIA).

(B) đúng vì X là C.

(C) đúng vì trong hợp chất Na2CO3 vừa có liên kết ion giữa Na+ và CO32- ; vừa có liên kết cộng hóa trị trong ion CO32-.

(D) sai vì oxit cao nhất của X với oxi là CO2 (oxit axit).


Câu 72:

Cho sơ đồ phản ứng: K2MnO4 + H2O → KMnO4 + MnO2 + KOH. Tỉ lệ số phân tử K2MnO4 bị oxi hóa và bị khử là
Xem đáp án

Phương pháp giải:

Trong phản ứng này Mn6+ vừa đóng vai trò chất khử – bị oxi hóa (lên Mn7+), vừa đóng vai trò chất oxi hóa – bị khử (xuống Mn4+).

Viết các quá trình nhường – nhận electron

Tỉ lệ số phân tử K2MnO4 bị oxi hóa và bị khử (không cần cân bằng PTHH).

Giải chi tiết:

Các quá trình nhường – nhận electron:

Mn6+ → Mn7+ + 1e | x2 (quá trình oxi hóa)

Mn6+ + 2e → Mn4+ | x1 (quá trình khử)

Tỉ lệ Mn6+ bị oxi hóa và Mn6+ bị khử hay tỉ lệ số phân tử K2MnO4 bị oxi hóa và bị khử là 2 : 1.


Câu 73:

Oxi hóa hoàn toàn 3,01 gam hợp chất hữu cơ X rồi dẫn sản phẩm lần lượt qua bình đựng H2SO4 đặc và bình đựng KOH dư thì thấy khối lượng các bình tăng lên tương ứng là 1,89 gam và 6,16 gam. Hãy xác định công thức phân tử của X biết tỉ khối hơi của X so với khí oxi bằng 2,6875. (Cho NTK: O=16, C=12, H=1).
Xem đáp án

Phương pháp giải:

- Khối lượng bình đựng H2SO4 đặc tăng là khối lượng của H2O, tính được số mol H2O, số mol H và khối lượng H.

- Khối lượng bình đựng KOH tăng là khối lượng của CO2, tính được số mol CO2, số mol C và khối lượng C.

- Xét tổng khối lượng C và khối lượng H với khối lượng của X, kết luận X có O trong phân tử hay không.

- Tính khối lượng và số mol O.

- Gọi công thức phân tử của X là \[{C_x}{H_y}{O_z}\].

\[x:y:z = {n_C}:{n_H}:{n_O}\]

Kết luận công thức đơn giản nhất của X.

- Dựa vào tỷ khối của X so với khí O2, tính phân tử khối của X.

- Kết luận công thức phân tử của X.

Giải chi tiết:

- Khối lượng bình đựng H2SO4 tăng chính là khối lượng của H2O.

\[ \to {m_{{H_2}O}} = 1,89{\mkern 1mu} gam \to {n_{{H_2}O}} = \frac{{1,89}}{{18}} = 0,105{\mkern 1mu} mol\]

\[ \to {n_H} = 2{n_{{H_2}O}} = 2.0,105 = 0,21{\mkern 1mu} mol \to {m_H} = 0,21.1 = 0,21{\mkern 1mu} gam\]

- Khối lượng bình đựng KOH tăng chính là khối lượng của CO2.

\[ \to {m_{C{O_2}}} = 6,16{\mkern 1mu} gam \to {n_{C{O_2}}} = \frac{{6,16}}{{44}} = 0,14{\mkern 1mu} mol\]

\[ \to {n_C} = {n_{C{O_2}}} = 0,14{\mkern 1mu} mol \to {m_C} = 0,14.12 = 1,68{\mkern 1mu} gam\]

Ta thấy: mC + mH = 1,68 + 0,21 = 1,89 < mX

→ Trong X có chứa O.

\[{m_O} = {m_X} - {m_C} - {m_H} = 3,01 - 1,68 - 0,21 = 1,12{\mkern 1mu} gam \to {n_O} = \frac{{1,12}}{{16}} = 0,07{\mkern 1mu} mol\]

Gọi công thức phân tử của X là CxHyOz

\[x:y:z = {n_C}:{n_H}:{n_O} = 0,14:0,21:0,07 = 2:3:1\]

Vậy công thức đơn giản nhất của X là C2H3O

Vì tỷ khối hơi của X so với khí oxi bằng 2,6875 \[ \to {M_X} = 2,6875.32 = 86\]

\[ \to (2.12 + 3.1 + 1.16)n = 86 \to n = 2\]

Vậy công thức phân tử của X là C4H6O2.


Câu 74:

Thủy phân hoàn toàn 1 mol peptit mạch hở X, thu được 2 mol Gly, 2 mol Ala và 1 mol Val. Mặt khác, thủy phân không hoàn toàn X, thu được hỗn hợp các amino axit và các peptit (trong đó có Gly-Ala-Val) nhưng không thu được peptit Gly-Gly. Số công thức cấu tạo phù hợp với tính chất của X là
Xem đáp án

Phương pháp giải:

- 1 phân tử X chứa 2 Gly, 2 Ala, 1 Val X là pentapeptit.

- X thủy phân không hoàn toàn tạo Gly-Ala-Val và không có Gly - Gly các CTCT thỏa mãn.

Giải chi tiết:

- 1 phân tử X chứa 2 Gly, 2 Ala, 1 Val X là pentapeptit.

- X thủy phân không hoàn toàn tạo Gly-Ala-Val và không có Gly-Gly nên các công thức cấu tạo thỏa mãn là:

     Gly-Ala-Val-Gly-Ala

     Gly-Ala-Val-Ala-Gly

     Ala-Gly -Ala-Val-Gly

     Gly-Ala-Gly -Ala-Val

Vậy có tất cả 4 công thức cấu tạo phù hợp với tính chất của X.


Câu 75:

Trong buổi hòa nhạc được tổ chức ở Nhà Hát lớn Hà Nội nhân dịp kỷ niệm 1010 năm Thăng Long. Một người ngồi dưới khán đài nghe được âm do một chiếc đàn giao hưởng phát ra có mức cường độ âm 1,2B. Khi dàn nhạc giao hưởng thực hiện bản hợp xướng người đó cảm nhận được âm là 2,376B. Hỏi dàn nhạc giao hưởng đó có bao nhiêu người?
Xem đáp án

Phương pháp giải:

Công thức tính mực cường độ âm: \[L = 10\log \frac{I}{{{I_0}}}{\mkern 1mu} \left( {dB} \right) = \log \frac{I}{{{I_0}}}{\mkern 1mu} \left( B \right)\]

Công thức tính cường độ âm: \[I = \frac{P}{S} = \frac{P}{{4\pi {R^2}}}\]

Giải chi tiết:

Khi có một đàn giao hưởng: \[{I_1} = \frac{P}{{4\pi {R^2}}} \Rightarrow {L_1} = \log \frac{{{I_1}}}{{{I_0}}} = 1,2B\]

Khi có n đàn giao hưởng: \[{I_2} = \frac{{nP}}{{4\pi {R^2}}} \Rightarrow {L_2} = \log \frac{{n{I_1}}}{{{I_0}}} = 2,376B\]

\[ \Rightarrow {L_2} - {L_1} = 1,176 \Leftrightarrow \log n = 1,176 \Rightarrow n = {10^{1,176}} = 15\]

Giàn nhạc giao hưởng có 15 người.


Câu 76:

Động năng dao động của một con lắc lò xo được mô tả theo thế năng dao động của nó bằng đồ thị như hình vẽ. Cho biết khối lượng của vật bằng 100g vật dao động giữa hai vị trí cách nhau 8cm. Tần số góc của dao động
Động năng dao động của một con lắc lò xo được mô tả theo thế năng dao động của nó bằng đồ thị như hình vẽ. Cho biết khối lượng của vật bằng 100g vật dao động giữa hai vị trí cách nhau 8cm. Tần số góc của dao động (ảnh 1)
Xem đáp án

Phương pháp giải:

Sử dụng kĩ năng đọc đồ thị

Độ dài quỹ đạo dao động: \[L = 2A\]

Thế năng của con lắc lò xo: \[{{\rm{W}}_t} = \frac{1}{2}m{\omega ^2}{x^2}\]

Giải chi tiết:

Độ dài quỹ đạo dao động của con lắc là: \[L = 2A \Rightarrow A = \frac{L}{2} = \frac{8}{2} = 4{\mkern 1mu} {\mkern 1mu} \left( {cm} \right) = 0,04{\mkern 1mu} {\mkern 1mu} \left( m \right)\]

Từ đồ thị ta thấy khi động năng bằng 0, thế năng của con lắc: \[{{\rm{W}}_{t\max }} = \frac{1}{2}m{\omega ^2}{A^2} = {4.10^{ - 3}}{\mkern 1mu} {\mkern 1mu} \left( J \right)\]

\[ \Rightarrow \frac{1}{2}0,1.{\omega ^2}{.0,04^2} = {4.10^{ - 3}} \Rightarrow \omega = 5\sqrt 2 {\mkern 1mu} {\mkern 1mu} \left( {rad/s} \right)\]


Câu 77:

Một đoạn mạch gồm điện trở thuần R, cuộn dây cảm thuần có độ tự cảm L, tụ điện có điện dung C thay đổi được mắc nối tiếp. Đặt vào hai đầu đoạn mạch một điện áp xoay chiều \[u = U\sqrt 2 .\cos \left( {100\pi t} \right)V\]. Khi \[C = {C_1}\] thì công suất tiêu thụ của mạch là \[P = 100W\] và cường độ dòng điện qua mạch có biểu thức \[i = {I_0}.\cos \left( {100\pi t + \frac{\pi }{3}} \right)A\]. Khi \[C = {C_2}\], công suất tiêu thụ của mạch đạt cực đại. Giá trị cực đại đó là:
Xem đáp án

Phương pháp giải:

Công suất tiêu thụ : \[P = {I^2}.R = \frac{{{U^2}.R}}{{{R^2} + {{({Z_L} - {Z_C})}^2}}}\]

Độ lệch pha giữa u và i được xác định : \[\tan \varphi = \frac{{{Z_L} - {Z_C}}}{R}\]

Thay đổi C để P cực đại thì tức là xảy ra cộng hưởng, khi đó \[{P_{\max }} = \frac{{{U^2}}}{R}\]

Giải chi tiết:

Khi C = C1 thì độ lệch pha giữa u và i được xác định:

\[\tan \varphi = \frac{{{Z_L} - {Z_{{C_1}}}}}{R} \Rightarrow \tan \frac{{ - \pi }}{3} = \frac{{{Z_L} - {Z_C}_1}}{R} = - \sqrt 3 \] \[ \Rightarrow {Z_L} - {Z_{{C_1}}} = - \sqrt 3 .R\]

Áp dụng công thức tính công suất:

\[P = {I^2}.R = \frac{{{U^2}.R}}{{{R^2} + {{({Z_L} - {Z_{{C_1}}})}^2}}} \Rightarrow 100 = \frac{{{U^2}.R}}{{{R^2} + {{({Z_L} - {Z_{C1}})}^2}}}\]

\[ \Rightarrow 100 = \frac{{{U^2}.R}}{{{R^2} + {{( - \sqrt 3 R)}^2}}} = \frac{{{U^2}}}{{4.R}}\]

Thay đổi C để P cực đại thì tức là xảy ra cộng hưởng, khi đó: \[{P_{\max }} = \frac{{{U^2}}}{R} = 4.\frac{{{U^2}}}{{4.R}} = 4.100 = 400{\rm{W}}\]


Câu 78:

Trong thí nghiệm Y-âng về giao thoa ánh sáng, trên màn quan sát thu được khoảng cách giữa 9 vân sáng liên tiếp là L. Tịnh tiến màn 36 cm theo phương vuông góc với màn tới vị trí mới thì khoảng cách giữa 11 vân sáng liên tiếp cũng là L. Khoảng cách giữa màn và mặt phẳng chứa hai khe lúc đầu là
Xem đáp án

Phương pháp giải:

+ Khoảng cách giữa n vân sáng liên tiếp: \[\left( {n - 1} \right)i\]

+ Sử dụng biểu thức tính khoảng vân: \[i = \frac{{\lambda D}}{a}\]

Giải chi tiết:

Ta có:

+ Khoảng cách giữa 9 vân sáng liên tiếp: \[8i = L \Rightarrow i = \frac{L}{8}\]

+ Khoảng cách giữa 11 vân sáng liên tiếp: \[10i' = L \Rightarrow i' = \frac{L}{{10}}\]

Lại có: \[\left\{ {\begin{array}{*{20}{l}}{i = \frac{{\lambda D}}{a}}\\{i' = \frac{{\lambda \left( {D - 0,36} \right)}}{a}}\end{array}} \right.\]

Lấy \[\frac{{i'}}{i} = \frac{{D - 0,36}}{D} = \frac{{\frac{L}{{10}}}}{{\frac{L}{8}}} = 0,8 \Rightarrow D = 1,8m\]


Câu 79:

Pha sáng quang hợp không tạo ra
Xem đáp án

Phương pháp giải:

Giải chi tiết:

Pha sáng quang hợp không tạo ra C­6H12O6, chất này được tạo ra ở pha tối.


Câu 80:

Trong cơ chế duy trì cân bằng nội môi, bộ phận điều khiển có vai trò
Xem đáp án

Phương pháp giải:

Giải chi tiết:

Trong cơ chế duy trì cân bằng nội môi, bộ phận điều khiển có vai trò gửi tín hiệu thần kinh hay hormon để điều khiển hoạt động của bộ phận thực hiện.


Câu 81:

Một loài thực vật có bộ nhiễm sắc thể 2n = 6. Trên mỗi cặp nhiễm sắc thể, xét một gen có ba alen, Do đột biến, trong loài đã xuất hiện ba dạng thể ba tương ứng với các cặp nhiễm sắc thể. Theo lí thuyết, các thể ba này có tối đa bao nhiêu loại kiểu gen về các gen đang xét?
Xem đáp án

Phương pháp giải:

Áp dụng công thức tính số kiểu gen tối đa trong quần thể (n là số alen)

Nếu gen nằm trên NST thường: \[\frac{{n(n + 1)}}{2}\] kiểu gen hay \[C_n^2 + n\]

Quần thể tam bội (3n): \[\frac{{r\left( {r + 1} \right)\left( {r + 2} \right)}}{{1.2.3}}\]  (có thể áp dụng cho thể ba)

Giải chi tiết:

Mỗi cặp NST bình thường có \[C_3^2 + 3 = 6\] kểu gen;

Cặp 3 nhiễm có \[\frac{{3\left( {3 + 1} \right)\left( {3 + 2} \right)}}{{1.2.3}} = 10\] KG

Có thể chọn được 3 dạng thể ba \[C_3^1\]

Số KG : \[C_3^1 \times {6^2} \times 10 = 1080\]


Câu 82:

Cho các biện pháp sau:

(1) Đưa thêm một gen lạ vào hệ gen.                      (2) Làm biến đổi một gen đã có sẵn trong hệ gen.

 (3) Gây đột biến đa bội ở cây trồng.                      (4) Cấy truyền phôi ở động vật.

Người ta có thể tạo ra sinh vật biến đổi gen bằng các biện pháp

Xem đáp án

Phương pháp giải:

Giải chi tiết:

Người ta có thể tạo ra sinh vật biến đổi gen bằng các biện pháp

(1) Đưa thêm một gen lạ vào hệ gen.

(2) Làm biến đổi một gen đã có sẵn trong hệ gen.

Loại bỏ hoặc làm bất hoạt một gen nào đó. (SGK Sinh 12 trang 84).


Câu 83:

Ý nghĩa về mặt văn hóa – xã hội của vị trí địa lý nước ta là
Xem đáp án

Phương pháp giải:

Kiến thức bài 2 – Vị trí địa lý, phạm vi lãnh thổ

Giải chi tiết:

Về mặt văn hóa – xã hội, vị trí liền kề cùng với nhiều nét tương đồng về lịch sử, văn hóa – xã hội và mối giao lưu lâu đời đã tạo điều kiện thuận lợi cho nước ta chung sống hòa bình, hợp tác hữu nghị và cùng phát triển với các nước láng giềng và các nước trong khu vực Đông Nam Á.


Câu 84:

Khó khăn lớn nhất về mặt tự nhiên đối với việc phát triển kinh tế - xã hội của vùng đồi núi là:
Xem đáp án

Phương pháp giải:

Kiến thức bài 6 – Đất nước nhiều đồi núi (liên hệ thế mạnh – hạn chế của vùng đồi núi)

Giải chi tiết:

Chú ý từ khóa “khó khăn lớn nhất” =>  nước ta có ¾ diện tích lãnh thổ là đồi núi, vùng đồi núi có địa hình hiểm trở, bị chia cắt mạnh, nhiều sông suối hẻm vực gây trở ngại cho giao thông đi lại, hạn ché sự  phát triển giao lưu kinh tế - xã hội trong vùng và giữa vùng đồi núi với đồng bằng.


Câu 85:

Ở miền khí hậu phía Bắc, trong mùa đông độ lạnh giảm dần về phía Tây vì
Xem đáp án

Phương pháp giải:

Kiến thức bài 8 – Thiên nhiên nhiệt đới ẩm gió mùa

Giải chi tiết:

Ở miền khí hậu phía Bắc, trong mùa đông độ lạnh giảm về phía Tây do bức chắn địa hình dãy Hoàng Liên Sơn đã ngăn cản ảnh hưởng của gió mùa Đông Bắc giảm dần về phía tây, khiến vùng núi Tây Bắc có mùa đông ngắn và bớt lạnh hơn so với Đông Bắc.


Câu 86:

Ở vùng đồi núi nước ta, sự phân hóa thiên nhiên theo Đông - Tây chủ yếu do
Xem đáp án

Phương pháp giải:

Kiến thức bài 10 – Thiên nhiên phân hóa đa dạng

Giải chi tiết:

Ở vùng đồi núi nước ta, sự phân hóa thiên nhiên theo Đông - Tây chủ yếu do tác động của gió mùa với hướng của các dãy núi chạy hướng Tây Bắc – Đông Nam. Cụ thể:

- Do tác động của gió mùa Đông Bắc kết hợp với dãy Hoàng Liên Sơn đã tạo nên sự phân hóa thiên nhiên giữa Tây Bắc và Đông Bắc: trong khi thiên nhiên vùng núi Đông Bắc mang sắc thái cận nhiệt đới gió mùa thì vùng núi thấp phía nam Tây Bắc lại có cảng quan thiên nhiên nhiệt đới gió mùa, vùng núi cao có cảnh quan ôn đới.

- Khi sườn Đông Trường Sơn đón gió từ biển vào tạo nên mùa mưa vào thu đông thì Tây Nguyên lại là mùa khô, nhiều nơi khô hạn gay gắt, xuất hiện cảnh quan rừng thưa. Còn Tây Nguyên vào mùa mưa thì bên sườn Đông Trường Sơn nhiều nơi chịu tác động của gió Tây khô nóng.


Câu 87:

Cơ quan nào của tổ chức Liên hợp quốc giữ vai trò trọng yếu trong việc duy trì hòa bình, an ninh thế giới?
Xem đáp án

Phương pháp giải:

SGK Lịch sử 12, trang 5

Giải chi tiết:

Hội đồng Bảo an của tổ chức Liên hợp quốc giữ vai trò trọng yếu trong việc duy trì hòa bình, an ninh thế giới.


Câu 88:

Điểm tương đồng trong công cuộc cải cách, mở cửa ở Trung Quốc với công cuộc cải tổ của Liên Xô và đổi mới đất nước ở Việt Nam là gì?
Xem đáp án

Phương pháp giải:

Dựa vào kiến thức đã học về công cuộc cải cách, mở cửa ở Trung Quốc (SGK Lịch sử 12, trang 23) với công cuộc cải tổ của Liên Xô (SGK Lịch sử 12, trang 14 - 15) và đổi mới đất nước ở Việt Nam (SGK Lịch sử 12, trang 208 - 209) để phân tích các phương án.

Giải chi tiết:

A loại vì cải cách ở 3 nước được tiến hành khi đã giành được độc lập.

B loại vì Việt Nam và Trung Quốc không tiến hành đa nguyên, đa đảng.

C chọn vì cả 3 nước đều tiến hành cải cách khi đất nước lâm vào tình trạng khủng hoảng kéo dài.

D loại vì đổi mới nhằm đưa đất nước thoát khỏi khủng hoảng, riêng ở Liên Xô thì thực hiện đa nguyên đa đảng nên vai trò của Đảng Cộng sản bị suy giảm, cũng là 1 trong những nguyên nhân làm cho công cuộc cải tổ thất bại, CNXH ở Liên Xô sụp đổ.


Câu 89:

Trong chiến lược Chiến tranh đặc biệt (1961 - 1965) ở miền Nam Việt Nam, Mỹ đã
Xem đáp án

Phương pháp giải:

SGK Lịch sử 12, trang 169.

Giải chi tiết:

Trong chiến lược Chiến tranh đặc biệt (1961 - 1965) miền Nam Việt Nam, Mỹ đã sử dụng các chiến thuật trực thăng vận và thiết xa vận.


Câu 90:

Cuộc đấu tranh bảo vệ nền độc lập của nhân dân Việt Nam (từ tháng 9-1945 đến tháng 12-1946) là một thành công về
Xem đáp án

Phương pháp giải:

Dựa vào kiến thức đã học về tình hình Việt Nam từ sau Cách mạng tháng Tám đến trước ngày 19/12/1946 để xác định được những khó khăn đưa nước ta vào tình thế ngàn cân treo sợi tóc và những việc làm của Chính phủ Việt Nam Dân chủ Cộng hòa để giải quyết những khó khăn này. Trên cơ sở đó, phân tích các đáp án để rút ra được đánh giá đúng về thành công trong cuộc đấu tranh bảo vệ nền độc lập của nhân dân Việt Nam (từ tháng 9-1945 đến tháng 12-1946).

Giải chi tiết:

A chọn vì ta chọn hòa hoãn với quân Trung Hoa Dân quốc để đánh Pháp nhưng sau khi Pháp và quân Trung Hoa Dân quốc đã hợp tác với nhau qua Hiệp ước Hoa – Pháp (28/2/1946) thì ta hòa hoãn với quân Trung Hoa Dân quốc để đuổi 20 vận quân Trung Hoa Dân quốc về nước và tạo thời gian hòa bình để chuẩn bị mọi mặt cho cuộc kháng chiến chống Pháp về sau. Đến khi không thể tiếp tục nhân nhượng với Pháp vì chúng gửi tối hậu thư yêu cầu ta giải tán lực lượng tự vệ chiến đấu và giao quyền kiểm soát thủ đô cho chúng (nếu tiếp tục nhân nhượng thì sẽ mất nước), Đảng và Chính phủ ta phát động quần chúng nhân dân cầm vũ khí chiến đấu để bảo vệ nền độc lập của mình.

B loại vì phải từ năm 1950 trở đi thì ta mới nhận được sự giúp đỡ về mọi mặt của các nước xã hội chủ nghĩa.

C loại vì mặt trận thống nhất dân tộc của ba nước Đông Dương được thành lập trong giai đoạn 1936 - 1939.

D loại vì ta thực hiện nhâ n nhượng có nguyên tắc với kẻ thù.

Trong các loại vật liệu xây dựng thì sắt, thép là một trong những vật liệu phổ biến nhất trên thế giới, được sử dụng làm vật liệu cho công trình xây dựng, đồ dùng hay trong ngành công nghiệp, cơ khí, ...

Các loại sắt, thép xây dựng được chế tạo thành các nhóm hợp kim khác nhau, tùy theo thành phần hóa học của các nguyên tố để tạo ra vật liệu phù hợp với mục đích sử dụng. Vật liệu sắt thép nhìn chung có nhiều ưu điểm vượt trội hơn những vật liệu truyền thống, tự nhiên như: gỗ, đất, đá,... nhờ chất lượng về độ cứng, độ đàn hồi, tính dễ uốn và độ bền cao. Sắt thép xây dựng được ứng dụng nhiều trong lĩnh vực đời sống và mang lại hiệu quả kinh tế cao.


Câu 91:

Luyện gang từ 10 tấn quặng hematit chứa 64% Fe2O3 thu được m tấn gang chứa 2,5% cacbon và tạp chất, hiệu suất của quá trình sản xuất là 85%. (Biết C = 12; O = 16; Fe = 56). Giá trị của m là
Xem đáp án

Phương pháp giải:

Bản chất của quá trình luyện gang: Fe2O3 → 2Fe

Dựa vào quá trình luyện gang → xác định được khối lượng gang (lý thuyết – chứa 97,5% Fe)

Do H = 85% khối lượng gang thực tế thu được (mTT = mLT.H).

Giải chi tiết:

Khối lượng Fe2O3 đem luyện gang là 10.64% = 6,4 (tấn)

Xét quá trình luyện gang:

Fe2O3 → 2Fe

 160        2.56  (tấn)

  6,4 →     x     (tấn)

\[{m_{Fe(LT)}} = x = \frac{{6,4.2.56}}{{160}} = 4,48\] (tấn)

Vì gang chứa 2,5% cacbon và tạp chất hay chứa 97,5% sắt

\[{m_{gang(LT)}} = \frac{x}{{97,5\% }} = 4,59\]  (tấn).

\[{m_{gang(TT)}} = {m_{gang(LT)}}.H = 4,59.85\% = 3,91\] (tấn). (Do H = 85%).

Vậy m = 3,91 (tấn).


Câu 92:

Nếu vật làm bằng hợp kim Fe – Zn bị ăn mòn điện hóa thì trong quá trình ăn mòn
Xem đáp án

Phương pháp giải:

Trong pin điện, kim loại nào có tính khử mạnh hơn sẽ đóng vai trò anot (cực âm) và bị oxi hóa.

Giải chi tiết:

Trong pin điện Fe – Zn thì Zn có tính khử mạnh hơn nên đóng vai trò là anot (cực âm) và bị oxi hóa.


Câu 93:

Cho các nhận định sau:

(1) Thép và gang đều là hợp kim của sắt với cacbon và một số nguyên tố khác, trong đó thép có hàm lượng cacbon thấp hơn nhiều so với gang.

(2) Thép thường được luyện từ quặng oxit sắt.

(3) Nguyên tắc sản xuất thép là làm giảm hàm lượng các tạp chất C, S, Mn,… có trong gang bằng cách khử các tạp chất đó thành oxit rồi biến thành xỉ và tách ra khỏi thép.

(4) Nguyên tắc sản xuất gang là khử quặng sắt oxit bằng than cốc trong lò cao.

(5) Gang giòn và cứng hơn thép.

Số nhận định đúng

Xem đáp án

Phương pháp giải:

Dựa vào kiến thức tổng hợp về gang và thép.

Giải chi tiết:

(2) sai vì gang thường được luyện tử quặng oxit sắt.

(3) sai vì nguyên tắc sản xuất thép là làm giảm hàm lượng các tạp chất C, S, Mn,… có trong gang bằng cách oxi hóa các tạp chất đó thành oxit rồi biến thành xỉ và tách ra khỏi thép.

Vậy có 3 nhận định đúng là (1), (4) và (5).


Câu 95:

Sau khi học xong bài Tinh bột, Lan được biết "Iot là thuốc thử của hồ tinh bột". Lúc nấu cơm, Lan đã chắt một ít nước cơm ra bát con, sau đó nhỏ vào đó vài giọt dung dịch iot thì không thấy màu xanh tím. Lan để chiếc bát đó đến hôm sau để mang đến lớp hỏi cô giáo, nhưng trước khi đi học nhìn vào bát nước cơm lại thấy có màu xanh tím. Em hãy giải thích giúp Lan.
Xem đáp án

Phương pháp giải:

Dựa vào kiến thức đã học về tinh bột.

Giải chi tiết:

Do ở điều kiện thường, tinh bột có cấu tạo mạch hở ở dạng xoắn có lỗ rỗng nên hấp phụ iot cho màu xanh tím, còn ở nhiệt độ cao cấu trúc xoắn duỗi ra nên không hấp phụ được iot.


Câu 96:

Phản ứng tổng hợp glucozo trong cây xanh cần được cung cấp năng lượng 2813 kJ cho mỗi mol glucozo tạo thành:

6CO2 + 6H2O + 2813 kJ → C6H12O6 + 6O2

Nếu trong 1 phút mỗi cm2 lá xanh nhận được khoảng 2,09 J năng lượng từ mặt trời nhưng chỉ có 10% được sử dụng vào phản ứng tổng hợp glucozo. Với 1 ngày nắng (từ 6 giờ - 17 giờ) diện tích lá xanh là 1 m2, lượng glucozo tổng hợp được là bao nhiêu?

Xem đáp án

Phương pháp giải:

- Tính thời gian chiếu sáng trong 1 ngày.

- Tính năng lượng mặt trời 1 cm2 lá xanh nhận được từ mặt trời trong 1 ngày.

- Tính năng lượng mặt trời 1 m2 = 10cm2 lá xanh nhận được từ mặt trời trong 1 ngày.

- Tính năng lượng sử dụng cho phản ứng tổng hợp glucozo.

- Tính số mol glucozo tổng hợp được.

- Tính khối lượng glucozo tổng hợp được.

Giải chi tiết:

- Thời gian chiếu sáng trong 1 ngày là: 17 - 6 = 11 (giờ) = 660 phút.

- Năng lượng mặt trời 1 cm2 lá xanh nhận được từ mặt trời trong 1 ngày là: 2,09.660 = 1379,4 (J).

- Năng lượng mặt trời 1 m2 = 10cm2 lá xanh nhận được từ mặt trời trong 1 ngày là: 1379,4.104 = 13794 (kJ).

- Năng lượng sử dụng cho phản ứng tổng hợp glucozo là: 13794.10% = 1379,4 (kJ).

- Số mol glucozo tổng hợp được là: 1379,4 : 2813 = 0,490366 mol.

- Khối lượng glucozo tổng hợp được là: 0,490366.180 = 88,2659 gam ≈ 88,3 gam.


Câu 97:

Dựa vào các thông tin được cung cấp dưới đây để trả lời các câu từ 97 đến 99

Có bao giờ bạn từng nghe thấy người xung quanh hỏi nhau: “Chiếc ô tô này bao nhiêu mã lực?” hay “Động cơ bao nhiêu mã lực?”

Mã lực là đơn vị được sử dụng phổ biến hiện nay để tính công suất động cơ nhưng không phải ai cũng hiểu mã lực là gì và bằng bao nhiêu nếu tính ra các đơn vị thường dùng.

Khái niệm mã lực được đưa ra đầu tiên bởi nhà khoa học người Scotland, Jame Watt. Mã lực đúng như tên gọi của nó chính là “sức ngựa” (viết tắt là HP - Horse Power) là một đơn vị cũ dùng để chỉ công suất. Nó được định nghĩa là công suất cần thiết để nâng một khối lượng 75 kg lên cao 1 mét trong thời gian 1giây hay \[1HP = 75kgm/s\]. Trong thực tế để chuyển đổi nhanh chóng giữa các đơn vị "mã lực" và "kW" (kilô watt), người ta hay dùng các hệ số tương đối như sau:

  \[1{\rm{ }}HP = 0,736{\rm{ }}kW\]; hoặc

 \[1{\mkern 1mu} kW = 1,36{\rm{ }}HP\] .

Để có hình ảnh trực quan hơn, mã lực cơ học được mô tả là công mà một chú ngựa bỏ ra để kéo 33 000 pound (1 pound = 454 gram) lên 1 foot (30,48 cm) trong thời gian 1 phút (minute).

Có bao giờ bạn từng nghe thấy người xung quanh hỏi nhau: “Chiếc ô tô này bao nhiêu mã lực?” hay “Động cơ bao nhiêu mã lực?”  Mã lực là đơn vị được sử dụng phổ biến hiện nay để tính công suất động cơ nhưng không phải ai cũng hiểu mã lực là gì và bằng bao nhiêu nếu tính ra các đơn vị thường dùng. (ảnh 1)

Một chiếc mô tô dung tích \[500c{m^3}\] công suất 95,2 mã lực tương ứng với:

Xem đáp án

Phương pháp giải:

Đổi đơn vị: \[1{\rm{ }}HP = 0,736{\rm{ }}kW\]hoặc \[1{\mkern 1mu} kW = 1,36{\rm{ }}HP\]

Giải chi tiết:

Công suất của mô tô: \[P = 95,2{\mkern 1mu} HP = 95,2.0,736 = 70kW\]


Câu 98:

Có các phát biểu dưới đây. Số phát biểu không đúng là:

1. Mã lực là đơn vị đo công suất

2. Mã lực tương đương với đơn vị đo điện năng kwh.

3. Công suất của một máy càng lớn thì khả năng sinh công của máy đó càng nhanh

4. Công suất của một ô tô càng lớn thì khả năng tăng tốc càng nhanh

5. Công suất của một ô tô càng lớn thì lực kéo của nó càng khỏe

Xem đáp án

Phương pháp giải:

+ Sử dụng kĩ năng khai thác thông tin từ đoạn văn.

+ Công thức tính công: \[A = P.t\]

+ Động cơ đốt trong bao gồm 2 thông số chính là công suất và mô-men xoắn. Mô-men xoắn chính là lực sinh ra khi xảy ra hiện tượng vật thể quay quanh trục và nó có nguồn gốc từ những thí nghiệm của nhà khoa học nổi tiếng Archimede về đòn bẩy. Khi có một lực tác động vào một vật khiến nó quay quanh một điểm, mô-men xoắn sẽ xuất hiện. Một chiếc xe có mô-men xoắn càng lớn thì lực quay của bánh xe càng mạnh. Đại lượng còn lại là công suất của động cơ, công suất tượng trưng cho khả năng đạt tốc độ nhanh hay chậm mà không tượng trưng cho lực quay hay sức mạnh của bánh xe.

Giải chi tiết:

+ Mã lực là đơn vị đo công suất 1 Đúng.

+ Mã lực tương đương với đơn vị đo công suất \[W{\mkern 1mu} \left( {kW} \right)\] 2 Sai.

+ Ta có \[A = P.t\], khi P càng lớn thì A càng lớn 3 Đúng.

+ Công suất của một ô tô càng lớn thì khả năng tăng tốc càng nhanh 4 Đúng; 5 Sai.

Có 3 phát biểu đúng và 2 phát biểu sai.


Câu 99:

Muốn bơm nước từ một giếng sâu 15m lên mặt đất người ta dùng một máy bơm nước có công suất 2HP, hiệu suất 50%. Tính lượng nước bơm được trong 1 giờ? Biết khối lượng riêng của nước là \[D = 1000kg/{m^3}\]\[1HP = 736W\]. Lấy \[g = 10m/{s^2}\].
Xem đáp án

Phương pháp giải:

Công thức tính công: \[A = P.t\]

Thế năng trọng trường: \[{W_t} = mgh\]

Hiệu suất: \[H = \frac{{{A_{ci}}}}{{{A_{tp}}}}.100\% \]

Công thức tính khối lượng riêng: \[D = \frac{m}{V}\]

Giải chi tiết:

Công suất của máy bơm là: \[P = 2HP = 2.736 = 1472W\]

Công của máy bơm thực hiện trong 1 giờ (công toàn phần) là: \[A = P.t = 1472.3600 = 5{\mkern 1mu} 299{\mkern 1mu} 200J\]

Công để lượng nước m(kg) lên độ cao h (công có ích): \[{A_{ci}} = mgh\]

Hiệu suất của máy bơm: \[H = \frac{{{A_{ci}}}}{A}.100\% = 50\% \Leftrightarrow \frac{{{A_{ci}}}}{A} = 0,5 \Rightarrow {A_{ci}} = 0,5A\] \[ \Leftrightarrow mgh = 0,5.A \Rightarrow m = \frac{{0,5.A}}{{gh}}\]

Khối lượng nước bơm lên được trong 1 giờ là: \[m = \frac{{0,5.5{\mkern 1mu} 299{\mkern 1mu} 200}}{{10.15}} = 17664kg\]

Thể tích nước bơm lên được trong 1 giờ: \[m = D.V \Rightarrow V = \frac{m}{D} = \frac{{17664}}{{1000}} = 17,664{\mkern 1mu} \left( {{m^3}} \right)\]

m=D.VV=mD=176641000=17,664(m3)m=D.VV=mD=176641000=17,664(m3)


Câu 100:

Dựa vào các thông tin được cung cấp dưới đây để trả lời các câu từ 100 đến 102

 Nhà máy thủy điện là nơi chuyển đổi sức nước (thủy năng) thành điện năng. Nước được tụ lại từ các đập nước với một thế năng lớn. Qua một hệ thống ống dẫn đến các tổ máy, năng lượng dòng chảy của nước được truyền tới tua-bin nước và làm quay tua-bin, tua-bin nước được nối với máy phát điện, nơi chúng được chuyển thành năng lượng điện và thoát ra bằng cửa thoát.

 Năng lượng điện từ nhà máy thủy điện là một dạng năng lượng tái sinh, năng lượng sạch vì không thải các khí có hại cho môi trường như các nhà máy điện khác.

 Tại Việt Nam vai trò của nhà máy thủy điện là rất quan trọng. Nhà máy thủy điện Hòa Bình là nguồn cung cấp điện chính cho đường dây điện cao thế 500kV Bắc-Nam. Nhà máy thủy điện Hòa Bình có một số thông tin sau:

Tọa độ: \[{20^0}48'30''B - {105^0}19'26''\]

Dung tích: \[1.600.000.000{\rm{ }}{m^2}{\rm{ }}\left( {{{5,7.10}^{10}}{\mkern 1mu} {\mkern 1mu} cu{\mkern 1mu} ft} \right)\]

Diện tích bề mặt: \[208k{m^2}{\rm{ }}\left( {80{\rm{ }}sq{\rm{ }}mi} \right)\]

Tua bin: \[8 \times 240{\rm{ }}MW\]

Công suất lắp đặt: \[1.920{\rm{ }}MW\]

Lượng điện hàng năm: \[8.160{\rm{ }}GWh\]

Ở các tổ máy phát điện của nhà máy thủy điện xảy ra quá trình biến đổi:

Xem đáp án

Phương pháp giải:

Thế năng trọng trường: \[{W_t} = mgh\]

Động năng: \[{W_d} = \frac{1}{2}m{v^2}\]

Cơ năng: \[W = {W_t} + {W_d} = mgh + \frac{1}{2}m{v^2}\]

Giải chi tiết:

Trong nhà máy thủy điện, thế năng của nước trong hồ chứa đã được chuyển hóa thành động năng, rồi thành điện năng.

Phát biểu đúng là: Ở các tổ máy phát điện của nhà máy thủy điện xảy ra quá trình biến đổi cơ năng thành điện năng.


Câu 101:

Số thông tin đúng trong số các thông tin về nhà máy thủy điện Hòa Bình dưới đây là:

1. Có vĩ độ \[{20^0}48'30''B - {105^0}19'26''\]

2. cu ftcu ft (foot khối) là đơn vị đo thể tích trong hệ đo lường Anh - Mỹ, với: \[1{\rm{ }}cu{\rm{ }}ft = 0,02807{m^3}\]

3. 8 tổ máy hoạt động hết công suất thì mỗi năm sản ra được lượng điện năng là \[8.160{\rm{ }}GWh\].

4. sq mi (dặm vuông Anh) là đơn vị đo diện tích trong hệ đo lường Anh - Mỹ, với: \[1{\rm{ }}sq{\rm{ }}mi \approx 2,6{m^2}\]

5. Công suất mỗi tổ máy theo thiết kế là 240 MW.

Xem đáp án

Phương pháp giải:

Xử lí thông tin từ đoạn văn.

Công thức tính điện năng: \[A = P.t\]

Giải chi tiết:

1. Nhà máy thủy điện Hòa Bình có tọa độ: \[{20^0}48'30''B - {105^0}19'26''\]

Có vĩ độ \[{20^0}48'30''B\] và kinh độ \[{105^0}19'26''\] 1 Đúng.

2. Dung tích:  \[1.600.000.000{\rm{ }}{m^2}{\rm{ }}\left( {5,7.10{\mkern 1mu} {\mkern 1mu} cu{\mkern 1mu} ft} \right)\]

\[ \Rightarrow V = 1{\mkern 1mu} {\mkern 1mu} 600{\mkern 1mu} {\mkern 1mu} 000{\mkern 1mu} {\mkern 1mu} 000{\rm{ }}{m^2} = {5,7.10^{10}}{\mkern 1mu} cu{\mkern 1mu} ft\]

\[ \Rightarrow 1{\mkern 1mu} cu{\mkern 1mu} ft = \frac{{1{\mkern 1mu} {\mkern 1mu} 600{\mkern 1mu} {\mkern 1mu} 000{\mkern 1mu} {\mkern 1mu} 000}}{{{{5,7.10}^{10}}}} \approx 0,02807{m^3}\] 2 Đúng.

3. Tua bin: \[8 \times 240{\rm{ }}MW\]×240 MW8×240 MW

Công suất hoạt động của 8 tổ máy là: \[P = 8.240 = 1920W = 1,920MW\]

Khi 8 tổ máy hoạt động hết công suất (8 tổ máy đều hoạt động trong 365 ngày, mỗi ngày 24 giờ) thì mỗi năm sản ra được lượng điện năng là: \[A = P.t = 1,920.365.24 = 16{\mkern 1mu} 819,2GWh\]

3 Sai.

4. Diện tích bề mặt: \[208k{m^2}{\rm{ }}\left( {80{\rm{ }}sq{\rm{ }}mi} \right)\]

\[ \Rightarrow S = 208k{m^2} = 80{\mkern 1mu} sq{\mkern 1mu} mi \Rightarrow 1sq{\mkern 1mu} mi = \frac{{208}}{{80}} = 2,6k{m^2}\]

4 Sai.

5. Tua bin: \[8 \times 240{\rm{ }}MW\] Có 8 tổ máy, công suất mỗi tổ máy theo thiết kế là 240 MW

5 Đúng.

Có 3 phát biểu đúng, 2 phát biểu sai.


Câu 102:

Một nhà máy thủy điện có công suất phát điện là \[{2.10^8}W\]có hiệu suất bằng 80. Mực nước ở hồ chứa nước có độ cao \[1000m\] so với tua bin của máy phát điện. Tính lưu lượng nước đến tua bin của máy phát điện \[\left( {{m^3}/s} \right)\]. Coi \[1{m^3}\] nước tương đương với \[{10^3}kg\]. Lấy \[g = 10m/{s^2}.\].
Xem đáp án

Phương pháp giải:

+ Thế năng trọng trường: \[{W_t} = mgh\]

+ Hiệu suất: \[H = \frac{{{A_{ci}}}}{{{A_{tp}}}}.100\% = \frac{{{P_{ci}}}}{{{P_{tp}}}}.100\% \]

Giải chi tiết:

+ Công suất phát điện: \[{P_{ci}} = {2.10^8}W\]

+ Thế năng của nước ở độ cao h chuyển hóa thành động năng của dòng nước trong tua bin (công toàn phần) và chuyển hóa thành công phát điện ở máy phát (công có ích).

Do đó, hiệu suất của nhà máy được tính: \[H = \frac{{{P_{ci}}}}{{{P_{tp}}}}.100\% \Leftrightarrow \frac{{{{2.10}^8}}}{{mgh}}.100\% = 80\% \] \[ \Leftrightarrow \frac{{{{2.10}^8}}}{{m.10.1000}} = 0,8\]

\[ \Rightarrow m = \frac{{{{2.10}^8}}}{{0,8.10.1000}} = 25000kg = {25.10^3}kg\]

Như vậy trong 1 giây có một khối lượng nước là \[m = {25.10^3}kg\] nước chảy qua ống, hay lưu lượng của nước trong ống là \[25{\mkern 1mu} \left( {{m^3}/s} \right)\]


Câu 103:

Dựa vào các thông tin được cung cấp dưới đây để trả lời các câu từ 103 đến 105

Bệnh tan máu bẩm sinh (Thalas - semia), viết tắt là Thal, là một bệnh do gen lặn (a) nằm trên nhiễm sắc thể thường gây nên. Người bị bệnh nhân 2 alen lặn từ bố và mẹ, biểu hiện bệnh ở dạng hồng cầu bị phá hủy quá mức dẫn đến tình trạng thiếu máu.

Theo thống kê (2001) người ta nhận thấy, bệnh Thal thường gặp ở các dân tộc vùng cao, vùng xa như: Thái, Mường, Tày, Ê đê, Khơ me, ít gặp ở người Kinh. Cụ thể, tỉ lệ mắc bệnh:

Nhóm 1: Người Mường, Thái, Tày là 25%;

Nhóm 2: Người Ê đê, Khơ me là 40%;

Nhóm 3: Người Kinh là 4%.

Với giả thiết là cấu trúc di truyền ban đầu của các dân tộc đều giống nhau, và ở dân tộc Kinh thì việc kết hôn hoàn toàn ngẫu nhiên.

Tỉ lệ mắc bệnh tan máu bẩm sinh ở các dân tộc vùng cao, vùng xa như: Thái, Mường, Tày, Ê đê, Khơ me cao là do

Xem đáp án

Phương pháp giải:

Giải chi tiết:

Ở các dân tộc thiểu số thường xảy ra kết hôn gần làm tăng tỉ lệ xuất hiện kiểu gen đồng hợp lặn gây bệnh tan máu bẩm sinh.


Câu 104:

Tần số alen gây bệnh trong cộng đồng người Kinh là
Xem đáp án

Phương pháp giải:

Giải chi tiết:

Cộng đồng người Kinh là kết hôn ngẫu nhiên nên cân bằng về di truyền → tỉ lệ người bị bệnh là 4% → aa = 4% → tần số alen a = \[\sqrt {0.04} \] = 0,2; A=0,8.


Câu 105:

Một cặp vợ chồng người Ê đê không mắc bệnh tan máu bẩm sinh nhưng sinh ra người con trai bị bệnh. Họ dự định sinh thêm 2 người con nữa. Xác suất họ sinh được 1 con trai và 1 con gái đều không bị bệnh là
Xem đáp án

Phương pháp giải:

Giải chi tiết:

Cặp vợ chồng này bình thường nhưng sinh con bị bệnh → đều mang gen gây bệnh.

A- không bị tan máu bẩm sinh

a- bị tan máu bẩm sinh

Cặp vợ chồng này có kiểu gen Aa.

Xác suất họ sinh được 1 con trai và một con gái là: \[\frac{1}{2} \times \frac{1}{2} \times 2 = \frac{1}{2}\]

Xác suất họ sinh 2 đứa con không bị bệnh là: \[\frac{3}{4} \times \frac{3}{4} = \frac{9}{{16}}\]

Vậy xác suất cần tính là: \[\frac{9}{{16}} \times \frac{1}{2} = \frac{9}{{32}}\]


Câu 106:

Dựa vào các thông tin được cung cấp dưới đây để trả lời các câu từ 106 đến 108

Nuôi cây tế bào thực vật in vitro tạo mô sẹo

Kĩ thuật nuôi cấy tế bao thực vật in vitro được hoàn thiện và phát triển nhờ tìm ta ra môi trường nuôi cây chuẩn kết hợp với việc sử dụng các hoocmon sinh trường như auxin, giberelin, xitokinin... Ngày nay, người ta có thể nuôi cấy nhiều loại tế bao của cây (chồi, lá, thân, rễ, hoa...) để tạo thành mô sẹo (mô gồm nhiều tế bào chưa biệt hoá, có khả năng sinh trưởng mạnh). Từ mô sẹo, điều khiển cho tế bao biệt hoá thành các mô khác nhau (rễ, thân, lá...) và tái sinh ra cây trưởng thành. Kĩ thuật này cho phép nhan nhanh các giống cây trồng có năng suất cao, chất lượng tốt, thich nghi với điều kiện sinh thái nhất định, chống chịu tốt với nhiều loại sâu, bệnh... Ví du, các nhà tạo giống Việt Nam đã thành công ở các cây như: khoai tây, mía, dứa. Mót số giống cây quý hiếm khác cũng được bảo tồn nguồn gen khỏi nguy cơ tuyệt chủng bằng phương pháp nuôi cấy tế bào.

Nếu trong môi trường nuôi cấy có nồng độ auxin/kinetin = 3/0,02 thì sẽ kích thích hình thành

Xem đáp án

Phương pháp giải:

Giải chi tiết:

Trong nuôi cấy mô, auxin có tác dụng kích thích hình thành rễ, kinetin kích thích hình thành chồi.

Nếu trong môi trường nuôi cấy có nồng độ auxin/kinetin = 3/0,02 thì sẽ kích thích hình thành rễ.


Câu 107:

Cơ sở tế bào học của nuôi cấy mô, tế bào được dựa trên
Xem đáp án

Phương pháp giải:

Giải chi tiết:

Cơ sở tế bào học của nuôi cấy mô, tế bào là sự sinh sản của tế bào – tức là nguyên phân


Câu 108:

Nuôi cấy các hạt phấn của một cây có kiểu gen AaBbDdee để tạo nên các mô đơn bội. Sau đó xử lí các mô đơn bội này bằng cônsixin để gây lưỡng bội hoá, thu được 80 cây lưỡng bội. Cho biết mỗi gen quy định một tính trạng, không xảy ra đột biến gen và đột biến cấu trúc NST. Theo lí thuyết, khi nói về 80 cây này, phát biểu nào sau đây đúng?
Xem đáp án

Phương pháp giải:

Giải chi tiết:

Khi lưỡng bội hóa bằng conxixin ta thu được các cá thể có kiểu gen thuần chủng, khi giảm phân chỉ tạo ra 1 loại giao tử.

A sai, do cây ban đầu có cặp gen ee nên cây con tạo ra không thể có cặp gen EE

B đúng

C sai, do chúng có kiểu gen khác nhau nên kiểu hình cũng khác nhau.

D sai, các cây này có tối đa 8 loại kiểu gen do cây ban đầu có thể tạo tối đa 8 loại giao tử.


Câu 109:

Dựa vào các thông tin được cung cấp dưới đây để trả lời các câu từ 109 đến 111

Nước ta có đường bờ biển dài 3260km chạy từ mũi Ngọc (Quảng Ninh) đến Hà Tiên (Kiên Giang), lại có nhiều vũng, vịnh rộng, kín gió và nhiều đảo, quần đảo ven bờ, nằm trên đường hàng hải quốc tế. Có thể nói, đây là điều kiện thích hợp cho việc phát triển giao thông đường biển. Ở mỗi vùng có những thế mạnh khác nhau. Bắc Bộ và Trung Bộ có biển Đông bao bọc với vịnh Bắc Bộ, các vũng vịnh đẹp nổi tiếng, tạo cơ sở để hình thành các hải cảng. Trong số này, cảng Cam Ranh (Khánh Hòa) được xếp vào một trong không nhiều cảng hàng đầu của thế giới về mặt tự nhiên. Ở Nam Bộ ba mặt giáp biển, cũng có nhiều vũng vịnh, đảo và quần đảo. Phía Tây trông ra vịnh Thái Lan rộng lớn...

Ở nước ta có nhiều tuyến đường biển trong nước và quốc tế. Các tuyến đường biển ven bờ chủ yếu là theo hướng bắc – nam. Quan trọng nhất là tuyến Hải Phòng – TP. Hồ Chí Minhh, dài 1500km. Các cảng biển và cụm cảng quan trọng là: Hải Phòng, Cái Lân, Đà Nẵng – Liên Chiểu – Chân Mây, Dung Quất, Nha Trang, Sài Gòn – Vũng Tàu – Thị Vải. Các tuyến đường biển quốc tế chủ yếu từ TP.Hồ Chí Minh và Hải Phòng tỏa đi các nơi và ngược lại.

Đối với giao thông vận tải biển, hệ thống cảng có ý nghĩa rất quan trọng. Trong tương lai, cần xây dựng hệ thống cảng biển để tạo ra các cửa ra – vào, đủ năng lực thông qua hàng hóa giữa nước ta với thế giới, hàng hóa quá cảnh cho các nước láng giềng và có cảng trung chuyển quốc tế, kết hợp cải tạo, hiện đại hóa hệ thống cảng biển hiện có với việc xây dựng một số cảng nước sâu ở cả ba miền.

(Nguồn: SGK Địa lí 12 cơ bản, trang 132 và Địa lí kinh tế - xã hội Việt Nam, trang 246 - 248)

Trả lời cho các câu 109, 110, 111 dưới đây:

Theo bài đọc, điều kiện cơ bản để phát triển giao thông đường biển nước ta là?

Xem đáp án

Phương pháp giải:

Đọc kĩ đoạn đầu tiên trong đoạn tư liệu

Giải chi tiết:

Điều kiện cơ bản để phát triển giao thông vận tải đường biển nước ta là ven biển có nhiều vũng, vịnh, kín gió, quần đảo ven bờ thuận lợi cho xây dựng các cảng biển; nước ta nằm trên các tuyến đường hàng hải hàng không quốc tế quan trọng dễ dàng trong giao lưu trao đổi với các nước bằng đường biển.


Câu 110:

Theo bài đọc, tuyến đường biển nội địa quan trọng nhất nước ta là
Xem đáp án

Phương pháp giải:

Đọc kĩ đoạn tư liệu số 1 

Giải chi tiết:

Tuyến đường biển ven bờ quan trọng nhất của nước ta là tuyến Hải Phòng – TP. Hồ Chí Minhh, dài 1500km.


Câu 111:

Theo bài đọc trên, biện pháp quan trọng để nâng cao năng lực vận chuyển các cảng biển là
Xem đáp án

Phương pháp giải:

Đọc kĩ đoạn tư liệu số 3

Giải chi tiết:

Biện pháp quan trọng để nâng cao năng lực vận chuyển các cảng biển là cần xây dựng cảng biển để tạo ra các cửa ra – vào, đủ năng lực thông qua hàng hóa giữ nước ta với thế giới, kết hợp cải tạo, hiện đại hóa heek thống cảng biển hiện có với việc xây dựng một số cảng nước sâu ở cả 3 miền.


Câu 112:

Việc chuyển dịch cơ cấu kinh tế có vai trò quan trọng ở đồng bằng sông Hồng. Xu hướng chung là tiếp tục giảm tỉ trọng của khu vực I (nông – lâm – ngư nghiệp), tăng nhanh tỉ trọng của khu vực II (công nghiệp – xây dựng) và khu vực III (dịch vụ) trên cơ sở đảm bảo tăng trường kinh tế với tốc độ nhanh, hiệu quả cao gắn với việc giải quyết các vấn đề xã hội và môi trường. Cho đến năm 2010, tỉ trọng của các khu vực tương ứng sẽ là 10%, 42% và 48%.

 Việc chuyển dịch cơ cấu kinh tế trong nội bộ từng ngành có sự khác nhau, nhưng trọng tâm là phát triển và hiện đại hóa công nghiệp chế biến, các ngành công nghiệp khác và dịch vụ gắn với yêu cầu phát triển nền nông nghiệp hàng hóa.

 Đối với khu vực I, giảm tỉ trọng của ngành trồng trọt, tăng tỉ trọng của ngành chăn nuôi và thủy sản. Riêng trong ngành trồng trọt lại giảm tỉ trọng của cây lương thực và tăng dần tỉ trọng của cây công nghiệp, cây thực phẩm, cây ăn quả.

 Đối với khu vực II, quá trình chuyển dịch gắn với việc hình thành các ngành công nghiệp trọng điểm để sử dụng có hiệu quả các thế mạnh về tự nhiên và con người của vùng. Đó là các ngành chế biến lương thực – thực phẩm, ngành dệt may và da giày, ngành sản xuất vật liệu xây dựng, ngành cơ khí – kĩ thuật, điện – điện tử.

 Đối với khu vực III, du lịch là một ngành tiềm năng. Đồng bằng sông Hồng có nhiều thế mạnh về du lịch, đặc biệt ở Hà Nội và vùng phụ cận cũng như ở Hải Phòng. Trong tương lai, du lịch sẽ có vị trí xứng đáng trong nền kinh tế của vùng. Các dịch vụ khác như tài chính, ngân hàng, giáo dục – đào tạo...cũng phát triển mạnh nhằm đẩy nhanh tốc độ chuyển dịch kinh tế.

 (Nguồn: SGK Địa lí 12 cơ bản, trang 131)

Xu hướng chuyển dịch cơ cấu kinh tế của đồng bằng sông Hồng là

Xem đáp án

Phương pháp giải:

Đọc kĩ đoạn tư liệu số 1

Giải chi tiết:

Xu hướng chuyển dịch cơ cấu kinh tế của đồng bằng sông Hồng là giảm tỉ trọng khu vực I, tăng tỉ trọng khu vực II và III.


Câu 113:

Đâu không phải là ngành công nghiệp trọng điểm của Đồng bằng sông Hồng?
Xem đáp án

Phương pháp giải:

Đọc kĩ đoạn tư liệu số 4

Giải chi tiết:

Các ngành công nghiệp trọng điểm của đồng bằng sông Hồng gồm: ngành chế biến lương thực – thực phẩm, ngành dệt may và da giày, ngành sản xuất vật liệu xây dựng, ngành cơ khí – kĩ thuật, điện – điện tử.

=> Công nghiệp năng lượng không phải công nghiệp trọng điểm của ĐBSH


Câu 114:

Đồng bằng sông Hồng có nhiều tiềm năng về du lịch, tập trung nhiều di tích, lễ hội, các làng nghề truyền thống là do
Xem đáp án

Phương pháp giải:

Liên hệ đặc điểm địa hình nước ta

Giải chi tiết:

Đồng bằng sông Hồng tập trung nhiều di tích, lễ hội, các làng nghề truyền thống là do vùng này có lịch sử khai phá lãnh thổ lâu đời gắn liền với truyền thống dựng nước và giữ nước hàng nghìn năm, mặt khác đây cũng là nơi có nền sản xuất lúa nước phát triển lâu đời.

=> Điều này đã để lại nhiều di tích lịch sử văn hóa, lễ hội truyền thống có giá trị


Câu 115:

Dựa vào các thông tin được cung cấp dưới đây để trả lời các câu hỏi từ câu 115 đến câu 117:

 Sau khi đến Quảng Châu, Nguyễn Ái Quốc mở lớp huấn luyện, đào tạo cán bộ. Phần lớn học viên là thanh niên, học sinh, trí thức Việt Nam yêu nước. Họ học làm cách mạng, học cách hoạt động bí mật. Phần lớn số học viên đó sau khi học xong, họ lại bí mật về nước truyền bá lí luận giải phóng dân tộc và tổ chức nhân dân”.

 Một số người được gửi sang học tại Trường Đại học Phương Đông ở Mátxcơva (Liên Xô) hoặc Trường Quân sự Hoàng Phố (Trung Quốc).

 Nguyễn Ái Quốc đã lựa chọn, giác ngộ một số thanh niên tích cực trong Tâm tâm xã, lập ra Cộng sản đoàn (2 - 1925.

 Tháng 6 – 1925, Nguyễn Ái Quốc thành lập Hội Việt Nam Cách mạng Thanh niên nhằm tổ chức và lãnh đạo quần chúng đoàn kết, tranh đấu để đánh đổ đế quốc chủ nghĩa Pháp và tay sai để tự cứu lấy mình. Cơ quan lãnh đạo cao nhất của Hội là Tổng bộ, trong đó có Nguyễn Ái Quốc, Hồ Tùng Mậu, Lê Hồng Sơn. Trụ sở của Tổng bộ đặt tại Quảng Châu.

 Báo Thanh niên của Hội do Nguyễn Ái Quốc sáng lập, ra số đầu tiên ngày 21 – 6 - 1925.

 Đầu năm 1927, tác phẩm Đường Kách mệnh, gồm những bài giảng của Nguyễn Ái Quốc ở các lớp huấn luyện tại Quảng Châu, được xuất bản.

 Báo Thanh niên và tác phẩm Đường Kách mệnh đã trang bị lí luận cách mạng giải phóng dân tộc cho cán bộ của Hội Việt Nam Cách mạng Thanh niên để tuyên truyền đến giai cấp công nhân và các tầng lớp nhân dân Việt Nam.

 Tại Quảng Châu, ngày 9 – 7 - 1925, Nguyễn Ái Quốc đã cùng một số nhà yêu nước Triều Tiên, Inđônêxia v.v. lập ra Hội Liên hiệp các dân tộc bị áp bức ở Á Đông. Tôn chỉ của Hội là liên lạc với các dân tộc bị áp bức để cùng làm cách mạng, đánh đổ đế quốc.

 Cuối năm 1928, thực hiện chủ trương “Vô sản hoá”, nhiều cán bộ của Hội Việt Nam Cách mạng Thanh niên đi vào các nhà máy, hầm mỏ, đồn điền, cùng sinh hoạt và lao động với công nhân để tuyên truyền vận động cách mạng, nâng cao ý thức chính trị cho giai cấp công nhân. Phong trào công nhân vì thế càng phát triển mạnh mẽ hơn và trở thành nòng cốt của phong trào dân tộc trong cả nước. Đấu tranh của công nhân đã nổ ra ở nhiều nơi.

(Nguồn: SGK Lịch sử 12, trang 83 – 84).

Tờ báo nào dưới đây là cơ quan ngôn luận của Hội Việt Nam Cách mạng Thanh niên?

Xem đáp án

Phương pháp giải:

Dựa vào thông tin được cung cấp để trả lời.

Giải chi tiết:

Tờ báo Thanh niên là cơ quan ngôn luận của Hội Việt Nam Cách mạng Thanh niên.


Câu 116:

Chủ trương “vô sản hóa” của Hội Việt Nam Cách mạng Thanh niên đã góp phần
Xem đáp án

Phương pháp giải:

Dựa vào thông tin được cung cấp để trả lời.

Giải chi tiết:

A loại vì phong trào diễn ra nhằm tuyên truyền vận động cách mạng, nâng cao ý thức chính trị cho giai cấp công nhân.

B loại vì Việt Nam Quốc dân đảng đi theo con đường dân chủ tư sản còn Hội Việt Nam Cách mạng Thanh niên đi theo con đường vô sản và phong trào diễn ra nhằm tuyên truyền vận động cách mạng, nâng cao ý thức chính trị cho giai cấp công nhân nên không thúc đẩy sự phân hóa của Việt Nam Quốc dân đảng.

C loại vì sự phân hóa là do Hội Việt Nam Cách mạng Thanh niên không thể tiếp tục lãnh đạo phong trào đấu tranh do hạn chế lịch sử của mình và do khuynh hướng vô sản đã cho thấy được sự phù hợp của mình đối với yêu cầu thực tế lịch sử Việt Nam đặt ra.

D chọn vì phong trào “vô sản hóa” của Hội Việt Nam Cách mạng thanh niên đã nâng cao ý thức chính trị cho giai cấp công nhân. Vì thế, phong trào công nhân ngày càng phát triển mạnh và trở thành nòng cốt của phong trào dân tộc trong cả nước, thúc đẩy phong trào công nhân chuyển từ tự phát sang tự giác.


Câu 117:

Ngày 9/7/1925, Nguyễn Ái Quốc cùng một số nhà yêu nước Triều Tiên, Inđônêxia...lập ra Hội Liên hiệp các dân tộc bị áp bức ở Á Đông đã chứng tỏ Người
Xem đáp án

Phương pháp giải:

Dựa vào thông tin được cung cấp để phân tích các phương án.

Giải chi tiết:

A chọn vì ngày 9/7/1925, Nguyễn Ái Quốc cùng một số nhà yêu nước Triều Tiên, Inđônêxia...lập ra Hội Liên hiệp các dân tộc bị áp bức ở Á Đông để cùng làm cách mạng, đánh đổ đế quốc, tức là Người đang tiếp tục tạo dựng mối quan hệ với cách mạng thế giới.

B loại vì lúc này Nguyễn Ái Quốc truyền bá lí luận cách mạng giải phóng dân tộc gián tiếp thông qua các sách báo gửi về trong nước, các bài viết trên báo Thanh niên, chứ chưa trực tiếp truyền bá được lí luận cách mạng giải phóng dân tộc.

C loại vì sự phân hóa của các tổ chức tiền cộng sản là do quá trình vận động giải phóng dân tộc và yêu cầu thực tế của lịch sử Việt Nam.

D loại vì sau khi tìm thấy con đường cứu nước đúng đắn cho dân tộc là con đường cách mạng vô sản thì Nguyễn Ái Quốc đã tích cực nghiên cứu chủ nghĩa Mác – Lê-nin và truyền bá lí luận giải phóng dân tộc về trong nước trong giai đoạn 1921 trở đi, chứ không phải đến tận năm 1925 mới bắt đầu xây dựng lí luận giải phóng dân tộc.


Câu 118:

Dựa vào các thông tin được cung cấp dưới đây để trả lời các câu hỏi từ câu 118 đến câu 120:

 Tháng 10 - 1929, khủng hoảng kinh tế bùng nổ ở Mĩ, sau đó lan ra toàn bộ thế giới tư bản, chấm dứt thời kì ổn định và tăng trưởng của chủ nghĩa tư bản. Cuộc khủng hoảng kéo dài gần 4 năm, trầm trọng nhất là năm 1932, chẳng những tàn phá nặng nề nền kinh tế các nước tư bản chủ nghĩa mà còn gây ra những hậu quả nghiêm trọng về chính trị, xã hội. Hàng chục triệu công nhân thất nghiệp, nông dân mất ruộng đất, sống trong cảnh nghèo đói, túng quẫn. Nhiều cuộc đấu tranh, biểu tình, tuần hành của những người thất nghiệp diễn ra ở khắp các nước.

 Khủng hoảng kinh tế đã đe doạ nghiêm trọng sự tồn tại của chủ nghĩa tư bản. Để cứu vãn tình thế, các nước tư bản buộc phải xem xét lại con đường phát triển của mình. Trong khi các nước Mĩ, Anh, Pháp tiến hành những cải cách kinh tế - xã hội để khắc phục hậu quả của cuộc khủng hoảng và đổi mới quá trình quản lí, tổ chức sản xuất thì các nước Đức, I-ta-li-a, Nhật Bản lại tìm kiếm lối thoát bằng những hình thức thống trị mới. Đó là việc thiết lập các chế độ độc tài phát xít - nên chuyên chính khủng bố công khai của những thế lực phản động nhất, hiếu chiến nhất.

 Đức, I-ta-li-a, Nhật Bản là những nước không có hoặc có ít thuộc địa, ngày càng thiếu vốn, thiểu nguyên liệu và thị trường, đã đi theo con đường phát xít hoá chế độ chính trị để cứu vãn tình trạng khủng hoảng nghiêm trọng của mình. Quan hệ giữa các cường quốc tự bản chuyển biến ngày càng phức tạp. Sự hình thành hai khối đế quốc đối lập : một bên là Mĩ, Anh, Pháp với một bên là Đức, I-ta-li-a, Nhật Bản và cuộc chạy đua vũ trang ráo riết đã báo hiệu nguy cơ của một cuộc chiến tranh thế giới mới.

(Nguồn: SGK Lịch sử 11, trang 61 – 62).

Cuộc khủng hoảng kinh tế thế giới (1929 - 1933) diễn ra đầu tiên ở đâu?

Xem đáp án

Phương pháp giải:

Dựa vào thông tin được cung cấp để trả lời.

Giải chi tiết:

Cuộc khủng hoảng kinh tế thế giới (1929 - 1933) diễn ra đầu tiên ở Mĩ.


Câu 119:

Để thoát khỏi khủng hoảng kinh tế 1929 -1933, các nước Đức, I-ta-li-a, Nhật Bản đã
Xem đáp án

Phương pháp giải:

Dựa vào thông tin được cung cấp để trả lời.

Giải chi tiết:

Để thoát khỏi khủng hoảng kinh tế 1929 -1933, các nước Đức, I-ta-li-a, Nhật Bản đã thiết lập chế độ độc tài phát xít.


Câu 120:

Hậu quả nặng nề nhất về kinh tế mà cuộc khủng hoảng 1929-1933 đem lại với các nước tư bản là gì?
Xem đáp án

Phương pháp giải:

Dựa vào thông tin được cung cấp, phân tích các phương án.

Giải chi tiết:

A chọn vì đây là hậu quả nặng nề nhất về kinh tế mà cuộc khủng hoảng 1929-1933 đem lại với các nước tư bản.

B loại vì đây là hậu quả về mặt xã hội.

C loại vì đây là hậu quả về chính trị.

D loại vì cuộc khủng hoảng không làm suy yếu, tan rã hệ thống thuộc địa.


Bắt đầu thi ngay